+7 495 120-13-73 | 8 800 500-97-74

(для регионов бесплатно)

Содержание

Сопротивление последовательной цепи формула — Морской флот

Последовательное соединение сопротивлений

Возьмем три постоянных сопротивления R1, R2 и R3 и включим их в цепь так, чтобы конец первого сопротивления R1 был соединен с началом второго сопротивления R 2, конец второго — с началом третьего R 3, а к началу первого сопротивления и к концу третьего подведем проводники от источника тока (рис. 1 ).

Такое соединение сопротивлений называется последовательным. Очевидно, что ток в такой цепи будет во всех ее точках один и тот же.

Рис 1 . Последовательное соединение сопротивлений

Как определить общее сопротивление цепи, если все включенные в нее последовательно сопротивления мы уже знаем? Используя положение, что напряжение U на зажимах источника тока равно сумме падений напряжений на участках цепи, мы можем написать:

U1 = IR1 U2 = IR2 и U3 = IR3

IR = IR1 + IR2 + IR3

Вынеся в правой части равенства I за скобки, получим IR = I(R1 + R2 + R3) .

Поделив теперь обе части равенства на I , будем окончательно иметь R = R1 + R2 + R3

Таким образом, мы пришли к выводу, что при последовательном соединении сопротивлений общее сопротивление всей цепи равно сумме сопротивлений отдельных участков.

Проверим этот вывод на следующем примере. Возьмем три постоянных сопротивления, величины которых известны (например, R1 == 10 Ом, R 2 = 20 Ом и R 3 = 50 Ом). Соединим их последовательно (рис. 2 ) и подключим к источнику тока, ЭДС которого равна 60 В (внутренним сопротивлением источника тока пренебрегаем).

Рис. 2. Пример последовательного соединения трех сопротивлений

Подсчитаем, какие показания должны дать приборы, включенные, как показано на схеме, если замкнуть цепь. Определим внешнее сопротивление цепи: R = 10 + 20 + 50 = 80 Ом.

Найдем ток в цепи по закону Ома: 60 / 80 = 0 ,75 А

Зная ток в цепи и сопротивления ее участков, определим падение напряжения на каждое участке цепи U 1 = 0,75 х 10 = 7,5 В, U 2 = 0,75 х 20=15 В, U3 = 0,75 х 50 = 37,5 В.

Зная падение напряжений на участках, определим общее падение напряжения во внешней цепи, т. е. напряжение на зажимах источника тока U = 7,5+15 + 37,5 = 60 В.

Мы получили таким образом, что U = 60 В, т. е. несуществующее равенство ЭДС источника тока и его напряжения. Объясняется это тем, что мы пренебрегли внутренним сопротивлением источника тока.

Замкнув теперь ключ выключатель К, можно убедиться по приборам, что наши подсчеты примерно верны.

Параллельное соединение сопротивлений

Возьмем два постоянных сопротивления R1 и R2 и соединим их так, чтобы начала этих сопротивлений были включены в одну общую точку а, а концы — в другую общую точку б. Соединив затем точки а и б с источником тока, получим замкнутую электрическую цепь. Такое соединение сопротивлений называется параллельным соединением.

Рис 3. Параллельное соединение сопротивлений

Проследим течение тока в этой цепи. От положительного полюса источника тока по соединительному проводнику ток дойдет до точки а. В точке а он разветвится, так как здесь сама цепь разветвляется на две отдельные ветви: первую ветвь с сопротивлением R1 и вторую — с сопротивлением R2. Обозначим токи в этих ветвях соответственно через I1 и I 2. Каждый из этих токов пойдет по своей ветви до точки б. В этой точке произойдет слияние токов в один общий ток, который и придет к отрицательному полюсу источника тока.

Таким образом, при параллельном соединении сопротивлений получается разветвленная цепь. Посмотрим, какое же будет соотношение между токами в составленной нами цепи.

Включим амперметр между положительным полюсом источника тока (+) и точкой а и заметим его показания. Включив затем амперметр (показанный «а рисунке пунктиром) в провод, соединяющий точку б с отрицательным полюсом источника тока (—), заметим, что прибор покажет ту же величину силы тока.

Значит, сила тока в цепи до ее разветвления (до точки а) равна силе тока после разветвления цепи (после точки б).

Будем теперь включать амперметр поочередно в каждую ветвь цепи, запоминая показания прибора. Пусть в первой ветви амперметр покажет силу тока I1 , а во второй — I 2. Сложив эти два показания амперметра, мы получим суммарный ток, по величине равный току I до разветвления (до точки а).

Следовательно, сила тока, протекающего до точки разветвления, равна сумме сил токов, утекающих от этой точки. I = I1 + I2 Выражая это формулой, получим

Это соотношение, имеющее большое практическое значение, носит название закона разветвленной цепи .

Рассмотрим теперь, каково будет соотношение между токами в ветвях.

Включим между точками а и б вольтметр и посмотрим, что он нам покажет. Во-первых, вольтметр покажет напряжение источника тока, так как он подключен, как это видно из рис. 3 , непосредственно к зажимам источника тока. Во-вторых, вольтметр покажет падения напряжений U1 и U2 на сопротивлениях R 1 и R2, так как он соединен с началом и концом каждого сопротивления.

Следовательно, при параллельном соединении сопротивлений напряжение на зажимах источника тока равно падению напряжения на каждом сопротивлении.

Это дает нам право написать, что U = U1 = U2 ,

где U — напряжение на зажимах источника тока; U 1 — падение напряжения на сопротивлении R 1 , U2 — падение напряжения на сопротивлении R2. Вспомним, что падение напряжения на участке цепи численно равно произведению силы тока, протекающего через этот участок, на сопротивление участка U = IR .

Поэтому для каждой ветви можно написать: U1 = I1R1 и U2 = I2R2 , но так как U 1 = U2, то и I1R1 = I2R2 .

Применяя к этому выражению правило пропорции, получим I1/ I2 = U2 / U1 т. е. ток в первой ветви будет во столько раз больше (или меньше) тока во второй ветви, во сколько раз сопротивление первой ветви меньше (или больше) сопротивления второй ветви.

Итак, мы пришли к важному выводу, заключающемуся в том, что при параллельном соединении сопротивлений общий ток цепи разветвляется на токи, обратно пропорциональные величинам сопротивлении параллельных ветвей. Иначе говоря, чем больше сопротивление ветви, тем меньший ток потечет через нее, и, наоборот, чем меньше сопротивление ветви, тем больший ток потечет через эту ветвь.

Убедимся в правильности этой зависимости на следующем примере. Соберем схему, состоящую из двух параллельно соединенных сопротивлений R1 и R 2, подключенных к источнику тока. Пусть R1 = 10 Ом, R2 = 20 Ом и U = 3 В.

Подсчитаем сначала, что покажет нам амперметр, включенный в каждую ветвь:

I1 = U / R1 = 3 / 10 = 0 ,3 А = 300 мА

I 2 = U / R 2 = 3 / 20 = 0,15 А = 150 мА

Общий ток в цепи I = I1 + I2 = 300 + 150 = 450 мА

Проделанный нами расчет подтверждает, что при параллельном соединении сопротивлений ток в цепи разветвляется обратно пропорционально сопротивлениям.

Действительно, R1 == 10 Ом вдвое меньше R 2 = 20 Ом, при этом I1 = 300 мА вдвое больше I2 = 150 мА. Общий ток в цепи I = 450 мА разветвился на две части так, что большая его часть ( I1 = 300 мА) пошла через меньшее сопротивление ( R1 = 10 Ом), а меньшая часть ( R2 = 150 мА) — через большее сопротивление ( R 2 = 20 Ом).

Такое разветвление тока в параллельных ветвях сходно с течением жидкости по трубам. Представьте себе трубу А, которая в каком-то месте разветвляется на две трубы Б и В различного диаметра (рис. 4). Так как диаметр трубы Б больше диаметра трубок В, то через трубу Б в одно и то же время пройдет больше воды, чем через трубу В, которая оказывает потоку воды большее сопротивление.

Рис. 4 . Через тонкую трубу в один и тот же промежуток времени пройдет воды меньше, чем через толстую

Рассмотрим теперь, чему будет равно общее сопротивление внешней цепи, состоящей из двух параллельно соединенных сопротивлений.

Под этим общим сопротивлением внешней цепи надо понимать такое сопротивление, которым можно было бы заменить при данном напряжении цепи оба параллельно включенных сопротивления, не изменяя при этом тока до разветвления. Такое сопротивление называется эквивалентным сопротивлением.

Вернемся к цепи, показанной на рис. 3, и посмотрим, чему будет равно эквивалентное сопротивление двух параллельно соединенных сопротивлений. Применяя к этой цепи закон Ома, мы можем написать: I = U/R , где I — ток во внешней цепи (до точки разветвления), U — напряжение внешней цепи, R — сопротивление внешней цепи, т. е. эквивалентное сопротивление.

Точно так же для каждой ветви I1 = U1 / R1 , I2 = U2 / R2 , где I1 и I 2 — токи в ветвях; U 1 и U2 — напряжение на ветвях; R1 и R2 — сопротивления ветвей.

По закону разветвленной цепи: I = I1 + I2

Подставляя значения токов, получим U / R = U1 / R1 + U2 / R2

Так как при параллельном соединении U = U1 = U2 , то можем написать U / R = U / R1 + U / R2

Вынеся U в правой части равенства за скобки, получим U / R = U (1 / R1 + 1 / R2 )

Разделив теперь обе части равенства на U , будем окончательно иметь 1 / R = 1 / R1 + 1 / R2

Помня, что проводимостью называется величина, обратная сопротивлению , мы можем сказать, что в полученной формуле 1 / R – проводимость внешней цепи; 1 / R1 проводимость первой ветви; 1 / R2- проводимость второй ветви.

На основании этой формулы делаем вывод: при параллельном соединении проводимость внешней цепи равна сумме проводимостей отдельных ветвей.

Следовательно, чтобы определить эквивалентное сопротивление включенных параллельно сопротивлений, надо определить проводимость цепи и взять величину, ей обратную.

Из формулы также следует, что проводимость цепи больше проводимости каждой ветви, а это значит, что эквивалентное сопротивление внешней цепи меньше наименьшего из включенных параллельно сопротивлений.

Рассматривая случай параллельного соединения сопротивлений, мы взяли наиболее простую цепь, состоящую из двух ветвей. Однако на практике могут встретиться случаи, когда цепь состоит из трех и более параллельных ветвей. Как же поступать в этих случаях?

Оказывается, все полученные нами соотношения остаются справедливыми и для цепи, состоящей из любого числа параллельно соединенных сопротивлений.

Чтобы убедиться в этом, рассмотрим следующий пример.

Возьмем три сопротивления R1 = 10 Ом, R2 = 20 Ом и R3 = 60 Ом и соединим их параллельно. Определим эквивалентное сопротивление цепи (рис. 5 ).

Рис. 5. Цепь с тремя параллельно соединенными сопротивлениями

Применяя для этой цепи формулу 1 / R = 1 / R1 + 1 / R2 , можем написать 1 / R = 1 / R1 + 1 / R2 + 1 / R3 и, подставляя известные величины, получим 1 / R = 1 / 10 + 1 / 20 + 1 / 60

Сложим эта дроби: 1/R = 10 / 60 = 1 / 6, т. е.. проводимость цепи 1 / R = 1 / 6 Следовательно, эквивалентное сопротивление R = 6 Ом.

Таким образом, эквивалентное сопротивление меньше наименьшего из включенных параллельно в цепь сопротивлений , т. е. меньше сопротивления R1.

Посмотрим теперь, действительно ли это сопротивление является эквивалентным, т. е. таким, которое могло бы заменить включенные параллельно сопротивления в 10, 20 и 60 Ом, не изменяя при этом силы тока до разветвления цепи.

Допустим, что напряжение внешней цепи, а следовательно, и напряжение на сопротивлениях R1, R2, R3 равно 12 В. Тогда сила токов в ветвях будет: I1 = U/R1 = 12 / 10 = 1 ,2 А I 2 = U/R 2 = 12 / 20 = 1 ,6 А I 3 = U/R1 = 12 / 60 = 0, 2 А

Общий ток в цепи получим, пользуясь формулой I = I1 + I2 + I3 = 1,2 + 0,6 + 0,2 = 2 А.

Проверим по формуле закона Ома, получится ли в цепи ток силой 2 А, если вместо трех параллельно включенных известных нам сопротивлений включено одно эквивалентное им сопротивление 6 Ом.

I = U / R = 12 / 6 = 2 А

Как видим, найденное нами сопротивление R = 6 Ом действительно является для данной цепи эквивалентным.

В этом можно убедиться и на измерительных приборах, если собрать схему с взятыми нами сопротивлениями, измерить ток во внешней цепи (до разветвления), затем заменить параллельно включенные сопротивления одним сопротивлением 6 Ом и снова измерить ток. Показания амперметра и в том и в другом случае будут примерно одинаковыми.

На практике могут встретиться также параллельные соединения, для которых рассчитать эквивалентное сопротивление можно проще, т. е. не определяя предварительно проводимостей, сразу найти сопротивление.

Например, если соединены параллельно два сопротивления R1 и R2 , то формулу 1 / R = 1 / R1 + 1 / R2 можно преобразовать так: 1/R = (R2 + R1) / R1 R2 и, решая равенство относительно R, получить R = R1 х R2 / ( R1 + R2 ), т. е. при параллельном соединении двух сопротивлений эквивалентное сопротивление цепи равно произведению включенных параллельно сопротивлений, деленному на их сумму.

В электрических цепях элементы могут соединяться по различным схемам, в том числе они имеют последовательное и параллельное соединение.

Последовательное соединение

При таком соединении проводники соединяются друг с другом последовательно, то есть, начало одного проводника будет соединяться с концом другого. Основная особенность данного соединения заключается в том, что все проводники принадлежат одному проводу, нет никаких разветвлений. Через каждый из проводников будет протекать один и тот же электрический ток. Но суммарное напряжение на проводниках будет равняться вместе взятым напряжениям на каждом из них.

Рассмотрим некоторое количество резисторов, соединенных последовательно. Так как нет разветвлений, то количество проходящего заряда через один проводник, будет равно количеству заряда, прошедшего через другой проводник. Силы тока на всех проводниках будут одинаковыми. Это основная особенность данного соединения.

Это соединение можно рассмотреть иначе. Все резисторы можно заменить одним эквивалентным резистором.

Ток на эквивалентном резисторе будет совпадать с общим током, протекающим через все резисторы. Эквивалентное общее напряжение будет складываться из напряжений на каждом резисторе. Это является разностью потенциалов на резисторе.

Если воспользоваться этими правилами и законом Ома, который подходит для каждого резистора, можно доказать, что сопротивление эквивалентного общего резистора будет равно сумме сопротивлений. Следствием первых двух правил будет являться третье правило.

Применение

Последовательное соединение используется, когда нужно целенаправленно включать или выключать какой-либо прибор, выключатель соединяют с ним по последовательной схеме. Например, электрический звонок будет звенеть только тогда, когда он будет последовательно соединен с источником и кнопкой. Согласно первому правилу, если электрический ток отсутствует хотя бы на одном из проводников, то его не будет и на других проводниках. И наоборот, если ток имеется хотя бы на одном проводнике, то он будет и на всех других проводниках. Также работает карманный фонарик, в котором есть кнопка, батарейка и лампочка. Все эти элементы необходимо соединить последовательно, так как нужно, чтобы фонарик светил, когда будет нажата кнопка.

Иногда последовательное соединение не приводит к нужным целям. Например, в квартире, где много люстр, лампочек и других устройств, не следует все лампы и устройства соединять последовательно, так как никогда не требуется одновременно включать свет в каждой из комнат квартиры. Для этого последовательное и параллельное соединение рассматривают отдельно, и для подключения осветительных приборов в квартире применяют параллельный вид схемы.

Параллельное соединение

В этом виде схемы все проводники соединяются параллельно друг с другом. Все начала проводников объединены в одну точку, и все концы также соединены вместе. Рассмотрим некоторое количество однородных проводников (резисторов), соединенных по параллельной схеме.

Этот вид соединения является разветвленным. В каждой ветви содержится по одному резистору. Электрический ток, дойдя до точки разветвления, разделяется на каждый резистор, и будет равняться сумме токов на всех сопротивлениях. Напряжение на всех элементах, соединенных параллельно, является одинаковым.

Все резисторы можно заменить одним эквивалентным резистором. Если воспользоваться законом Ома, можно получить выражение сопротивления. Если при последовательном соединении сопротивления складывались, то при параллельном будут складываться величины обратные им, как записано в формуле выше.

Применение

Если рассматривать соединения в бытовых условиях, то в квартире лампы освещения, люстры должны быть соединены параллельно. Если их соединить последовательно, то при включении одной лампочки мы включим все остальные. При параллельном же соединении мы можем, добавляя соответствующий выключатель в каждую из ветвей, включать соответствующую лампочку по мере желания. При этом такое включение одной лампы не влияет на остальные лампы.

Все электрические бытовые устройства в квартире соединены параллельно в сеть с напряжением 220 В, и подключены к распределительному щитку. Другими словами, параллельное соединение используется при необходимости подключения электрических устройств независимо друг от друга. Последовательное и параллельное соединение имеют свои особенности. Существуют также смешанные соединения.

Работа тока

Последовательное и параллельное соединение, рассмотренное ранее, было справедливо для величин напряжения, сопротивления и силы тока, являющихся основными. Работа тока определяется по формуле:

А = I х U х t, где А – работа тока, t – время течения по проводнику.

Для определения работы при последовательной схеме соединения, необходимо заменить в первоначальном выражении напряжение. Получаем:

А=I х (U1 + U2) х t

Раскрываем скобки и получаем, что на всей схеме работа определяется суммой на каждой нагрузке.

Точно также рассматриваем параллельную схему соединения. Только меняем уже не напряжение, а силу тока. Получается результат:

А = А1+А2

Мощность тока

При рассмотрении формулы мощности участка цепи снова необходимо пользоваться формулой:

Р=U х I

После аналогичных рассуждений выходит результат, что последовательное и параллельное соединение можно определить следующей формулой мощности:

Р=Р1 + Р2

Другими словами, при любых схемах общая мощность равна сумме всех мощностей в схеме. Этим можно объяснить, что не рекомендуется включать в квартире сразу несколько мощных электрических устройств, так как проводка может не выдержать такой мощности.

Влияние схемы соединения на новогоднюю гирлянду

После перегорания одной лампы в гирлянде можно определить вид схемы соединения. Если схема последовательная, то не будет гореть ни одной лампочки, так как сгоревшая лампочка разрывает общую цепь. Чтобы выяснить, какая именно лампочка сгорела, нужно проверять все подряд. Далее, заменить неисправную лампу, гирлянда будет функционировать.

При применении параллельной схемы соединения гирлянда будет продолжать работать, даже если одна или несколько ламп сгорели, так как цепь не разорвана полностью, а только один небольшой параллельный участок. Для восстановления такой гирлянды достаточно увидеть, какие лампы не горят, и заменить их.

Последовательное и параллельное соединение для конденсаторов

При последовательной схеме возникает такая картина: заряды от положительного полюса источника питания идут только на наружные пластины крайних конденсаторов. Конденсаторы, находящиеся между ними, передают заряд по цепи. Этим объясняется появление на всех пластинах равных зарядов с разными знаками. Исходя из этого, заряд любого конденсатора, соединенного по последовательной схеме, можно выразить такой формулой:

qобщ= q1 = q2 = q3

Для определения напряжения на любом конденсаторе, необходима формула:

U= q/С

Где С — емкость. Суммарное напряжение выражается таким же законом, который подходит для сопротивлений. Поэтому получаем формулу емкости:

С= q/(U1 + U2 + U3)

Чтобы сделать эту формулу проще, можно перевернуть дроби и заменить отношение разности потенциалов к заряду емкости. В результате получаем:

1/С= 1/С1 + 1/С2 + 1/C3

Немного иначе рассчитывается параллельное соединение конденсаторов.

Общий заряд вычисляется как сумма всех зарядов, накопившихся на пластинах всех конденсаторов. А величина напряжения также вычисляется по общим законам. В связи с этим формула суммарной емкости при параллельной схеме соединения выглядит так:

С= (q1 + q2 + q3)/U

Это значение рассчитывается как сумма каждого прибора в схеме:

С=С1 + С2 + С3

Смешанное соединение проводников

В электрической схеме участки цепи могут иметь и последовательное и параллельное соединение, переплетающихся между собой. Но все законы, рассмотренные выше для отдельных видов соединений, справедливы по-прежнему, и используются по этапам.

Сначала нужно мысленно разложить схему на отдельные части. Для лучшего представления ее рисуют на бумаге. Рассмотрим наш пример по изображенной выше схеме.

Удобнее всего ее изобразить, начиная с точек Б и В. Они расставляются на некотором расстоянии между собой и от края листа бумаги. С левой стороны к точке Б подключается один провод, а справа отходят два провода. Точка В наоборот, слева имеет две ветки, а после точки отходит один провод.

Далее нужно изобразить пространство между точками. По верхнему проводнику расположены 3 сопротивления с условными значениями 2, 3, 4. Снизу будет идти ток с индексом 5. Первые 3 сопротивления включены в схему последовательно, а пятый резистор подключен параллельно.

Остальные два сопротивления (первый и шестой) подключены последовательно с рассматриваемым нами участком Б-В. Поэтому схему дополняем 2-мя прямоугольниками по сторонам от выбранных точек.

Теперь используем формулу расчета сопротивления:

  • Первая формула для последовательного вида соединения.
  • Далее, для параллельной схемы.
  • И окончательно для последовательной схемы.

Аналогичным образом можно разложить на отдельные схемы любую сложную схему, включая соединения не только проводников в виде сопротивлений, но и конденсаторов. Чтобы научиться владеть приемами расчета по разным видам схем, необходимо потренироваться на практике, выполнив несколько заданий.

Проводники в электрических цепях могут соединяться последовательно и параллельно.

При последовательном соединении проводников (рис. 1.9.1) сила тока во всех проводниках одинакова:

Рисунок 1.9.1.

По закону Ома, напряжения и на проводниках равны

Общее напряжение на обоих проводниках равно сумме напряжений 1 и 2:

где – электрическое сопротивление всей цепи. Отсюда следует:

При последовательном соединении полное сопротивление цепи равно сумме сопротивлений отдельных проводников.

Этот результат справедлив для любого числа последовательно соединенных проводников.

При параллельном соединении (рис. 1.9.2) напряжения 1 и 2 на обоих проводниках одинаковы:

Сумма токов 1 + 2, протекающих по обоим проводникам, равна току в неразветвленной цепи:

Этот результат следует из того, что в точках разветвления токов (узлы и ) в цепи постоянного тока не могут накапливаться заряды. Например, к узлу за время Δ подтекает заряд Δ, а утекает от узла за то же время заряд 1Δ + 2Δ. Следовательно, = 1 + 2.

Рисунок 1.9.2.

Записывая на основании закона Ома

где – электрическое сопротивление всей цепи, получим

При параллельном соединении проводников величина, обратная общему сопротивлению цепи, равна сумме величин, обратных сопротивлениям параллельно включенных проводников.

Этот результат справедлив для любого числа параллельно включенных проводников.

Формулы для последовательного и параллельного соединения проводников позволяют во многих случаях рассчитывать сопротивление сложной цепи, состоящей из многих резисторов. На рис. 1.9.3 приведен пример такой сложной цепи и указана последовательность вычислений.

Рисунок 1.9.3.

Следует отметить, что далеко не все сложные цепи, состоящие из проводников с различными сопротивлениями, могут быть рассчитаны с помощью формул для последовательного и параллельного соединения. На рис. 1.9.4 приведен пример электрической цепи, которую нельзя рассчитать указанным выше методом.

Рисунок 1.9.4.

Цепи, подобные изображенной на рис. 1.9.4, а также цепи с разветвлениями, содержащие несколько источников, рассчитываются с помощью правил Кирхгофа.

Последовательное соединение проводников формула. Последовательное и параллельное соединения проводников

Содержание:

Как известно, соединение любого элемента схемы, независимо от его назначения, может быть двух видов — параллельное подключение и последовательное. Также возможно и смешанное, то есть последовательно параллельное соединение. Все зависит от назначения компонента и выполняемой им функции. А значит, и резисторы не избежали этих правил. Последовательное и параллельное сопротивление резисторов это по сути то же самое, что и параллельное и последовательное подключение источников света. В параллельной цепи схема подключения подразумевает вход на все резисторы из одной точки, а выход из другой. Попробуем разобраться, каким образом выполняется последовательное соединение, а каким — параллельное. И главное, в чем состоит разница между подобными соединениями и в каких случаях необходимо последовательное, а в каких параллельное соединение. Также интересен и расчет таких параметров, как общее напряжение и общее сопротивление цепи в случаях последовательного либо параллельного соединения. Начать следует с определений и правил.

Способы подключения и их особенности

Виды соединения потребителей или элементов играют очень важную роль, ведь именно от этого зависят характеристики всей схемы, параметры отдельных цепей и тому подобное. Для начала попробуем разобраться с последовательным подключением элементов к схеме.

Последовательное соединение

Последовательное подключение — это такое соединение, где резисторы (равно, как и другие потребители или элементы схем) подключаются друг за другом, при этом выход предыдущего подключается на вход следующего. Подобный вид коммутации элементов дает показатель, равный сумме сопротивлений этих элементов схемы. То есть если r1 = 4 Ом, а r2 = 6 Ом, то при подключении их в последовательную цепь, общее сопротивление составит 10 Ом. Если мы добавим последовательно еще один резистор на 5 Ом, сложение этих цифр даст 15 Ом — это и будет общее сопротивление последовательной цепи. То есть общие значения равны сумме всех сопротивлений. При его расчете для элементов, которые подключены последовательно, никаких вопросов не возникает — все просто и ясно. Именно поэтому не стоит даже останавливаться более серьезно на этой.

Совершенно по другим формулам и правилам производится расчет общего сопротивления резисторов при параллельном подключении, вот на нем имеет смысл остановиться поподробнее.

Параллельное соединение

Параллельным называется соединение, при котором все входы резисторов объединены в одной точке, а все выходы — во второй. Здесь главное понять, что общее сопротивление при подобном подключении будет всегда ниже, чем тот же параметр резистора, имеющего наименьшее.

Имеет смысл разобрать подобную особенность на примере, тогда понять это будет намного проще. Существует два резистора по 16 Ом, но при этом для правильного монтажа схемы требуется лишь 8 Ом. В данном случае при задействовании их обеих, при их параллельном включении в схему, как раз и получатся необходимые 8 Ом. Попробуем понять, по какой формуле возможны вычисления. Рассчитать этот параметр можно так: 1/Rобщ = 1/R1+1/R2, причем при добавлении элементов сумма может продолжаться до бесконечности.

Попробуем еще один пример. Параллельно соединены 2 резистора, с сопротивлением 4 и 10 Ом. Тогда общее будет равно 1/4 + 1/10, что будет равным 1:(0.25 + 0.1) = 1:0.35 = 2.85 Ом. Как видим, хотя резисторы и имели значительное сопротивление, при подключении их параллельнообщий показатель стал намного ниже.

Так же можно рассчитать общее сопротивление четырех параллельно подключенных резисторов, с номиналом 4, 5, 2 и 10 Ом. Вычисления, согласно формуле, будут такими: 1/Rобщ = 1/4+1/5+1/2+1/10, что будет равным 1:(0.25+0.2+0.5+0.1)=1/1.5 = 0.7 Ом.

Что же касается тока, протекающего через параллельно соединенные резисторы, то здесь необходимо обратиться к закону Кирхгофа, который гласит «сила тока при параллельном соединении, выходящего из цепи, равна току, входящему в цепь». А потому здесь законы физики решают все за нас. При этом общие показатели тока разделяются на значения, которые являются обратно пропорциональными сопротивлению ветки. Если сказать проще, то чем больше показатель сопротивления, тем меньшие токи будут проходить через этот резистор, но в общем, все же ток входа будет и на выходе. При параллельном соединении напряжение также остается на выходе таким же, как и на входе.

Схема параллельного соединения указана ниже.

Последовательно-параллельное соединение

Последовательно-параллельное соединение — это когда схема последовательного соединения содержит в себе параллельные сопротивления. В таком случае общее последовательное сопротивление будет равно сумме отдельно взятых общих параллельных. Метод вычислений одинаковый в соответствующих случаях.

Подведем итог

Подводя итог всему вышеизложенному можно сделать следующие выводы:

  1. При последовательном соединении резисторов не требуется особых формул для расчета общего сопротивления. Необходимо лишь сложить все показатели резисторов — сумма и будет общим сопротивлением.
  2. При параллельном соединении резисторов, общее сопротивление высчитывается по формуле 1/Rобщ = 1/R1+1/R2…+Rn.
  3. Эквивалентное сопротивление при параллельном соединении всегда меньше минимального подобного показателя одного из резисторов, входящих в схему.
  4. Ток, равно как и напряжение в параллельном соединении остается неизменным, то есть напряжение при последовательном соединении равно как на входе, так и на выходе.
  5. Последовательно-параллельное соединение при подсчетах подчиняется тем же законам.

В любом случае, каким бы ни было подключение, необходимо четко рассчитывать все показатели элементов, ведь параметры имеют очень важную роль при монтаже схем. И если ошибиться в них, то либо схема не будет работать, либо ее элементы просто сгорят от перегрузки. По сути, это правило применимо к любым схемам, даже в электромонтаже. Ведь провод по сечению подбирают также исходя из мощности и напряжения. А если поставить лампочку номиналом в 110 вольт в цепь с напряжением 220, несложно понять, что она моментально сгорит. Так же и с элементами радиоэлектроники. А потому — внимательность и скрупулезность в расчетах — залог правильной работы схемы.

Содержание:

Течение тока в электрической цепи осуществляется по проводникам, в направлении от источника к потребителям. В большинстве подобных схем используются медные провода и электрические приемники в заданном количестве, обладающие различным сопротивлением.

В зависимости выполняемых задач, в электрических цепях используется последовательное и параллельное соединение проводников. В некоторых случаях могут быть применены оба типа соединений, тогда этот вариант будет называться смешанным. Каждая схема имеет свои особенности и отличия, поэтому их нужно обязательно заранее учитывать при проектировании цепей, ремонте и обслуживании электрооборудования.

Последовательное соединение проводников

В электротехнике большое значение имеет последовательное и параллельное соединение проводников в электрической цепи. Среди них часто используется схема последовательного соединения проводников предполагающая такое же соединение потребителей. В этом случае включение в цепь выполняется друг за другом в порядке очередности. То есть, начало одного потребителя соединяется с концом другого при помощи проводов, без каких-либо ответвлений.

Свойства такой электрической цепи можно рассмотреть на примере участков цепи с двумя нагрузками. Силу тока, напряжение и сопротивление на каждом из них следует обозначить соответственно, как I1, U1, R1 и I2, U2, R2. В результате, получились соотношения, выражающие зависимость между величинами следующим образом: I = I1 = I2, U = U1 + U2, R = R1 + R2. Полученные данные подтверждаются практическим путем с помощью проведения измерений амперметром и вольтметром соответствующих участков.

Таким образом, последовательное соединение проводников отличается следующими индивидуальными особенностями:

  • Сила тока на всех участках цепи будет одинаковой.
  • Общее напряжение цепи составляет сумму напряжений на каждом участке.
  • Общее сопротивление включает в себя сопротивления каждого отдельного проводника.

Данные соотношения подходят для любого количества проводников, соединенных последовательно. Значение общего сопротивления всегда выше, чем сопротивление любого отдельно взятого проводника. Это связано с увеличением их общей длины при последовательном соединении, что приводит и к росту сопротивления.

Если соединить последовательно одинаковые элементы в количестве n, то получится R = n х R1, где R — общее сопротивление, R1 — сопротивление одного элемента, а n — количество элементов. Напряжение U, наоборот, делится на равные части, каждая из которых в n раз меньше общего значения. Например, если в сеть с напряжением 220 вольт последовательно включаются 10 ламп одинаковой мощности, то напряжение в любой из них составит: U1 = U/10 = 22 вольта.

Проводники, соединенные последовательно, имеют характерную отличительную особенность. Если во время работы отказал хотя-бы один из них, то течение тока прекращается во всей цепи. Наиболее ярким примером является , когда одна перегоревшая лампочка в последовательной цепи, приводит к выходу из строя всей системы. Для установления перегоревшей лампочки понадобится проверка всей гирлянды.

Параллельное соединение проводников

В электрических сетях проводники могут соединяться различными способами: последовательно, параллельно и комбинированно. Среди них параллельное соединение это такой вариант, когда проводники в начальных и конечных точках соединяются между собой. Таким образом, начала и концы нагрузок соединяются вместе, а сами нагрузки располагаются параллельно относительно друг друга. В электрической цепи могут содержаться два, три и более проводников, соединенных параллельно.

Если рассматривать последовательное и параллельное соединение, сила тока в последнем варианте может быть исследована с помощью следующей схемы. Берутся две лампы накаливания, обладающие одинаковым сопротивлением и соединенные параллельно. Для контроля к каждой лампочке подключается собственный . Кроме того, используется еще один амперметр, контролирующий общую силу тока в цепи. Проверочная схема дополняется источником питания и ключом.

После замыкания ключа нужно контролировать показания измерительных приборов. Амперметр на лампе № 1 покажет силу тока I1, а на лампе № 2 — силу тока I2. Общий амперметр показывает значение силы тока, равное сумме токов отдельно взятых, параллельно соединенных цепей: I = I1 + I2. В отличие от последовательного соединения, при перегорании одной из лампочек, другая будет нормально функционировать. Поэтому в домашних электрических сетях используется параллельное подключение приборов.

С помощью такой же схемы можно установить значение эквивалентного сопротивления. С этой целью в электрическую цепь добавляется вольтметр. Это позволяет измерить напряжение при параллельном соединении, сила тока при этом остается такой же. Здесь также имеются точки пересечения проводников, соединяющих обе лампы.

В результате измерений общее напряжение при параллельном соединении составит: U = U1 = U2. После этого можно рассчитать эквивалентное сопротивление, условно заменяющее все элементы, находящиеся в данной цепи. При параллельном соединении, в соответствии с законом Ома I = U/R, получается следующая формула: U/R = U1/R1 + U2/R2, в которой R является эквивалентным сопротивлением, R1 и R2 — сопротивления обеих лампочек, U = U1 = U2 — значение напряжения, показываемое вольтметром.

Следует учитывать и тот фактор, что токи в каждой цепи, в сумме составляют общую силу тока всей цепи. В окончательном виде формула, отражающая эквивалентное сопротивление будет выглядеть следующим образом: 1/R = 1/R1 + 1/R2. При увеличении количества элементов в таких цепях — увеличивается и число слагаемых в формуле. Различие в основных параметрах отличают друг от друга и источников тока, позволяя использовать их в различных электрических схемах.

Параллельное соединение проводников характеризуется достаточно малым значением эквивалентного сопротивления, поэтому сила тока будет сравнительно высокой. Данный фактор следует учитывать, когда в розетки включается большое количество электроприборов. В этом случае сила тока значительно возрастает, приводя к перегреву кабельных линий и последующим возгораниям.

Законы последовательного и параллельного соединения проводников

Данные законы, касающиеся обоих видов соединений проводников, частично уже были рассмотрены ранее.

Для более четкого их понимания и восприятия в практической плоскости, последовательное и параллельное соединение проводников, формулы следует рассматривать в определенной последовательности:

  • Последовательное соединение предполагает одинаковую силу тока в каждом проводнике: I = I1 = I2.
  • параллельное и последовательное соединение проводников объясняет в каждом случае по-своему. Например, при последовательном соединении, напряжения на всех проводниках будут равны между собой: U1 = IR1, U2 = IR2. Кроме того, при последовательном соединении напряжение составляет сумму напряжений каждого проводника: U = U1 + U2 = I(R1 + R2) = IR.
  • Полное сопротивление цепи при последовательном соединении состоит из суммы сопротивлений всех отдельно взятых проводников, независимо от их количества.
  • При параллельном соединении напряжение всей цепи равно напряжению на каждом из проводников: U1 = U2 = U.
  • Общая сила тока, измеренная во всей цепи, равна сумме токов, протекающих по всем проводникам, соединенных параллельно между собой: I = I1 + I2.

Для того чтобы более эффективно проектировать электрические сети, нужно хорошо знать последовательное и параллельное соединение проводников и его законы, находя им наиболее рациональное практическое применение.

Смешанное соединение проводников

В электрических сетях как правило используется последовательное параллельное и смешанное соединение проводников, предназначенное для конкретных условий эксплуатации. Однако чаще всего предпочтение отдается третьему варианту, представляющему собой совокупность комбинаций, состоящих из различных типов соединений.

В таких смешанных схемах активно применяется последовательное и параллельное соединение проводников, плюсы и минусы которых обязательно учитываются при проектировании электрических сетей. Эти соединения состоят не только из отдельно взятых резисторов, но и довольно сложных участков, включающих в себя множество элементов.

Смешанное соединение рассчитывается в соответствии с известными свойствами последовательного и параллельного соединения. Метод расчета заключается в разбивке схемы на более простые составные части, которые считаются отдельно, а потом суммируются друг с другом.

Ток в электроцепи проходит по проводникам от источника напряжения к нагрузке, то есть к лампам, приборам. В большинстве случаев в качестве проводника используются медные провода. В цепи может быть предусмотрено несколько элементов с разными сопротивлениями. В схеме приборов проводники могут быть соединены параллельно или последовательно, также могут быть смешанные типы.

Элемент схемы с сопротивлением называется резистором, напряжение данного элемента является разницей потенциалов между концами резистора. Параллельное и последовательное электрическое соединение проводников характеризуется единым принципом функционирования, согласно которому ток протекает от плюса к минусу, соответственно потенциал уменьшается. На электросхемах сопротивление проводки берется за 0, поскольку оно ничтожно низкое.

Параллельное соединение предполагает, что элементы цепы подсоединены к источнику параллельно и включаются одновременно. Последовательное соединение означает, что проводники сопротивления подключаются в строгой последовательности друг за другом.

При просчете используется метод идеализации, что существенно упрощает понимание. Фактически в электрических цепях потенциал постепенно снижается в процессе перемещения по проводке и элементам, которые входят в параллельное или последовательное соединение.

Последовательное соединение проводников

Схема последовательного соединения подразумевает, что они включаются в определенной последовательности один за другим. Причем сила тока во всех из них равна. Данные элементы создают на участке суммарное напряжение. Заряды не накапливаются в узлах электроцепи, поскольку в противном случае наблюдалось бы изменение напряжения и силы тока. При постоянном напряжении ток определяется значением сопротивления цепи, поэтому при последовательной схеме сопротивление меняется в случае изменения одной нагрузки.

Недостатком такой схемы является тот факт, что в случае выхода из строя одного элемента остальные также утрачивают возможность функционировать, поскольку цепь разрывается. Примером может служить гирлянда, которая не работает в случае перегорания одной лампочки. Это является ключевым отличием от параллельного соединения, в котором элементы могут функционировать по отдельности.

Последовательная схема предполагает, что по причине одноуровневого подключения проводников их сопротивление в любой точки сети равно. Общее сопротивление равняется сумме уменьшения напряжений отдельных элементов сети.

При данном типе соединения начало одного проводника подсоединяется к концу другого. Ключевая особенность соединения состоит в том, что все проводники находятся на одном проводе без разветвлений, и через каждый из них протекает один электроток. Однако общее напряжение равно сумме напряжений на каждом. Также можно рассмотреть соединение с другой точки зрения – все проводники заменяются одним эквивалентным резистором, и ток на нем совпадает с общим током, который проходит через все резисторы. Эквивалентное совокупное напряжение является суммой значений напряжения по каждому резистору. Так проявляется разность потенциалов на резисторе.

Использование последовательного подключения целесообразно, когда требуется специально включать и выключать определенное устройство. К примеру, электрозвонок может звенеть только в момент, когда присутствует соединение с источником напряжения и кнопкой. Первое правило гласит, что если тока нет хотя бы на одном из элементов цепи, то и на остальных его не будет. Соответственно при наличии тока в одном проводнике он есть и в остальных. Другим примером может служить фонарик на батарейках, который светит только при наличии батарейки, исправной лампочки и нажатой кнопки.

В некоторых случаях последовательная схема нецелесообразна. В квартире, где система освещения состоит из множества светильников, бра, люстр, не стоит организовывать схему такого типа, поскольку нет необходимости включать и выключать освещение во всех комнатах одновременно. С этой целью лучше использовать параллельное соединение, чтобы иметь возможность включения света в отдельно взятых комнатах.

Параллельное соединение проводников

В параллельной схеме проводники представляют собой набор резисторов, одни концы которых собираются в один узел, а другие – во второй узел. Предполагается, что напряжение в параллельном типе соединения одинаковое на всех участках цепи. Параллельные участки электроцепи носят название ветвей и проходят между двумя соединительными узлами, на них имеется одинаковое напряжение. Такое напряжение равно значению на каждом проводнике. Сумма показателей, обратных сопротивлениям ветвей, является обратной и по отношению к сопротивлению отдельного участка цепи параллельной схемы.

При параллельном и последовательном соединениях отличается система расчета сопротивлений отдельных проводников. В случае параллельной схемы ток уходит по ветвям, что способствует повышению проводимости цепи и уменьшает совокупное сопротивление. При параллельном подключении нескольких резисторов с аналогичными значениями совокупное сопротивление такой электроцепи будет меньше одного резистора число раз, равное числу .

В каждой ветви предусмотрено по одному резистору, и электроток при достижении точки разветвления делится и расходится к каждому резистору, его итоговое значение равно сумме токов на всех сопротивлениях. Все резисторы заменяются одним эквивалентным резистором. Применяя закон Ома, становится понятным значение сопротивления – при параллельной схеме суммируются значения, обратные сопротивлениям на резисторах.

При данной схеме значение тока обратно пропорционально значению сопротивления. Токи в резисторах не взаимосвязаны, поэтому при отключении одного из них это никоим образом не отразится на остальных. По этой причине такая схема используется во множестве устройств.

Рассматривая возможности применения параллельной схемы в быту, целесообразно отметить систему освещения квартиры. Все лампы и люстры должны быть соединены параллельно, в таком случае включение и отключение одного из них никак не влияет на работу остальных ламп. Таким образом, добавляя выключатель каждой лампочки в ветвь цепи, можно включать и отключать соответствующий светильник по необходимости. Все остальные лампы работают независимо.

Все электроприборы объединяются параллельно в электросеть с напряжением 220 В, затем они подключаются к . То есть все приборы подключаются независимо от подключения прочих устройств.

Законы последовательного и параллельного соединения проводников

Для детального понимания на практике обоих типов соединений, приведем формулы, объясняющие законы данных типов соединений. Расчет мощности при параллельном и последовательном типе соединения отличается.

При последовательной схеме имеется одинаковая сила тока во всех проводниках:

Согласно закону Ома, данные типы соединений проводников в разных случаях объясняются иначе. Так, в случае последовательной схемы, напряжения равны друг другу:

U1 = IR1, U2 = IR2.

Помимо этого, общее напряжение равно сумме напряжений отдельно взятых проводников:

U = U1 + U2 = I(R1 + R2) = IR.

Полное сопротивление электроцепи рассчитывается как сумма активных сопротивлений всех проводников, вне зависимости от их числа.

В случае параллельной схемы совокупное напряжение цепи аналогично напряжению отдельных элементов:

А совокупная сила электротока рассчитывается как сумма токов, которые имеются по всем проводникам, расположенным параллельно:

Чтобы обеспечить максимальную эффективность электрических сетей, необходимо понимать суть обоих типов соединений и применять их целесообразно, используя законы и рассчитывая рациональность практической реализации.

Смешанное соединение проводников

Последовательная и параллельная схема соединения сопротивления могут сочетаться в одной электросхеме при необходимости. К примеру, допускается подключение параллельных резисторов по последовательной или их группе, такое тип считается комбинированным или смешанным.

В таком случае совокупное сопротивление рассчитывается посредством получения сумм значений для параллельного соединения в системе и для последовательного. Сначала необходимо рассчитывать эквивалентные сопротивления резисторов в последовательной схеме, а затем элементов параллельного. Последовательное соединение считается приоритетным, причем схемы такого комбинированного типа часто используются в бытовой технике и приборах.

Итак, рассматривая типы подключений проводников в электроцепях и основываясь на законах их функционирования, можно полностью понять суть организации схем большинства бытовых электроприборов. При параллельном и последовательном соединениях расчет показателей сопротивления и силы тока отличается. Зная принципы расчета и формулы, можно грамотно использовать каждый тип организации цепей для подключения элементов оптимальным способом и с максимальной эффективностью.

Последовательное соединение сопротивлений

Возьмем три неизменных сопротивления R1, R2 и R3 и включим их в цепь так, чтоб конец первого сопротивления R1 был соединен с началом второго сопротивления R 2, конец второго — с началом третьего R 3, а к началу первого сопротивления и к концу третьего подведем проводники от источника тока (рис. 1 ).

Такое соединение сопротивлений именуется поочередным. Разумеется, что ток в таковой цепи будет во всех ее точках один и тот же.

Рис 1 . Последовательное соединение сопротивлений

Как найти общее сопротивление цепи, если все включенные в нее поочередно сопротивления мы уже знаем? Используя положение, что напряжение U на зажимах источника тока равно сумме падений напряжений на участках цепи, мы можем написать:

U = U1 + U2 + U3

где

U1 = IR1 U2 = IR2 и U3 = IR3

либо

IR = IR1 + IR2 + IR3

Вынеся в правой части равенства I за скобки, получим IR = I(R1 + R2 + R3) .

Поделив сейчас обе части равенства на I , будем совсем иметь R = R1 + R2 + R3

Таким макаром, мы сделали вывод, что при поочередном соединении сопротивлений общее сопротивление всей цепи равно сумме сопротивлений отдельных участков.

Проверим этот вывод на последующем примере. Возьмем три неизменных сопротивления, величины которых известны (к примеру, R1 == 10 Ом, R 2 = 20 Ом и R 3 = 50 Ом). Соединим их поочередно (рис. 2 ) и подключим к источнику тока, ЭДС которого равна 60 В (внутренним сопротивлением источника тока пренебрегаем).

Рис. 2. Пример поочередного соединения 3-х сопротивлений

Подсчитаем, какие показания должны дать приборы, включенные, как показано на схеме, если замкнуть цепь. Определим наружное сопротивление цепи: R = 10 + 20 + 50 = 80 Ом.

Найдем ток в цепи по закону Ома: 60 / 80 = 0 ,75 А

Зная ток в цепи и сопротивления ее участков, определим падение напряжения на каждое участке цепи U 1 = 0,75х 10 = 7,5 В, U 2 = 0,75 х 20=15 В, U3 = 0,75 х 50 = 37,5 В.

Зная падение напряжений на участках, определим общее падение напряжения во наружной цепи, т. е. напряжение на зажимах источника тока U = 7,5+15 + 37,5 = 60 В.

Мы получили таким макаром, что U = 60 В, т. е. несуществующее равенство ЭДС источника тока и его напряжения. Разъясняется это тем, что мы пренебрегли внутренним сопротивлением источника тока.

Замкнув сейчас ключ выключатель К, можно убедиться по устройствам, что наши подсчеты приблизительно верны.

Возьмем два неизменных сопротивления R1 и R2 и соединим их так, чтоб начала этих сопротивлений были включены в одну общую точку а, а концы — в другую общую точку б. Соединив потом точки а и б с источником тока, получим замкнутую электронную цепь. Такое соединение сопротивлений именуется параллельным соединением.

Рис 3. Параллельное соединение сопротивлений

Проследим течение тока в этой цепи. От положительного полюса источника тока по соединительному проводнику ток дойдет до точки а. В точке а он разветвится, потому что тут сама цепь разветвляется на две отдельные ветки: первую ветвь с сопротивлением R1 и вторую — с сопротивлением R2. Обозначим токи в этих ветвях соответственно через I1 и I 2. Любой из этих токов пойдет по собственной ветки до точки б. В этой точке произойдет слияние токов в один общий ток, который и придет к отрицательному полюсу источника тока.

Таким макаром, при параллельном соединении сопротивлений выходит разветвленная цепь. Поглядим, какое же будет соотношение меж токами в составленной нами цепи.

Включим амперметр меж положительным полюсом источника тока (+) и точкой а и заметим его показания. Включив потом амперметр (показанный «а рисунке пунктиром) в провод, соединяющий точку б с отрицательным полюсом источника тока (-), заметим, что прибор покажет ту же величину силы тока.

Означает, сила тока в цепи до ее разветвления (до точки а) равна силе тока после разветвления цепи (после точки б).

Будем сейчас включать амперметр попеременно в каждую ветвь цепи, запоминая показания прибора. Пусть в первой ветки амперметр покажет силу тока I1 , а во 2-ой — I 2. Сложив эти два показания амперметра, мы получим суммарный ток, по величине равный току I до разветвления (до точки а).

Как следует, сила тока, протекающего до точки разветвления, равна сумме сил токов, утекающих от этой точки. I = I1 + I2 Выражая это формулой, получим

Это соотношение, имеющее огромное практическое значение, носит заглавие закона разветвленной цепи .

Разглядим сейчас, каково будет соотношение меж токами в ветвях.

Включим меж точками а и б вольтметр и поглядим, что он нам покажет. Во-1-х, вольтметр покажет напряжение источника тока, потому что он подключен, как это видно из рис. 3 , конкретно к зажимам источника тока. Во-2-х, вольтметр покажет падения напряжений U1 и U2 на сопротивлениях R1 и R2, потому что он соединен с началом и концом каждого сопротивления.

Как следует, при параллельном соединении сопротивлений напряжение на зажимах источника тока равно падению напряжения на каждом сопротивлении.

Это дает нам право написать, что U = U1 = U2 ,

где U — напряжение на зажимах источника тока; U1 — падение напряжения на сопротивлении R1 , U2 — падение напряжения на сопротивлении R2. Вспомним, что падение напряжения на участке цепи численно равно произведению силы тока, протекающего через этот участок, на сопротивление участка U = IR .

Потому для каждой ветки можно написать: U1 = I1R1 и U2 = I2R2 , но потому что U1 = U2, то и I1R1 = I2R2 .

Применяя к этому выражению правило пропорции, получим I1/ I2 = U2 / U1 т. е. ток в первой ветки будет во столько раз больше (либо меньше) тока во 2-ой ветки, во сколько раз сопротивление первой ветки меньше (либо больше) сопротивления 2-ой ветки.

Итак, мы пришли к принципиальному выводу, заключающемуся в том, что при параллельном соединении сопротивлений общий ток цепи разветвляется на токи, назад пропорциональные величинам сопротивлении параллельных веток. По другому говоря, чем больше сопротивление ветки, тем наименьший ток потечет через нее, и, напротив, чем меньше сопротивление ветки, тем больший ток потечет через эту ветвь.

Убедимся в корректности этой зависимости на последующем примере. Соберем схему, состоящую из 2-ух параллельно соединенных сопротивлений R1 и R 2, присоединенных к источнику тока. Пусть R1 = 10 Ом, R2 = 20 Ом и U = 3 В.

Подсчитаем поначалу, что покажет нам амперметр, включенный в каждую ветвь:

I1 = U / R1 = 3 / 10 = 0 ,3 А = 300 мА

I 2 = U / R 2 = 3 / 20 = 0,15 А = 150 мА

Общий ток в цепи I = I1 +I2 = 300 + 150 = 450 мА

Проделанный нами расчет подтверждает, что при параллельном соединении сопротивлений ток в цепи разветвляется назад пропорционально сопротивлениям.

Вправду, R1 == 10 Ом в два раза меньше R 2 = 20 Ом, при всем этом I1 = 300 мА в два раза больше I2 = 150 мА. Общий ток в цепи I = 450 мА разветвился на две части так, что большая его часть (I1 = 300 мА) пошла через наименьшее сопротивление (R1 = 10 Ом), а наименьшая часть (R2 = 150 мА) -через большее сопротивление (R 2 = 20 Ом).

Такое разветвление тока в параллельных ветвях сходно с течением воды по трубам. Представьте для себя трубу А, которая в каком-то месте разветвляется на две трубы Б и В различного поперечника (рис. 4). Потому что поперечник трубы Б больше поперечника трубок В, то через трубу Б в одно и то же время пройдет больше воды, чем через трубу В, которая оказывает сгустку воды большее сопротивление.

Рис. 4

Разглядим сейчас, чему будет равно общее сопротивление наружной цепи, состоящей из 2-ух параллельно соединенных сопротивлений.

Под этим общим сопротивлением наружной цепи нужно осознавать такое сопротивление, которым можно было бы поменять при данном напряжении цепи оба параллельно включенных сопротивления, не изменяя при всем этом тока до разветвления. Такое сопротивление именуется эквивалентным сопротивлением.

Вернемся к цепи, показанной на рис. 3, и поглядим, чему будет равно эквивалентное сопротивление 2-ух параллельно соединенных сопротивлений. Применяя к этой цепи закон Ома, мы можем написать: I = U/R , где I — ток во наружной цепи (до точки разветвления), U — напряжение наружной цепи, R — сопротивление наружной цепи, т. е. эквивалентное сопротивление.

Точно так же для каждой ветки I1 = U1 / R1 , I2 = U2 / R2 , где I1 и I 2 — токи в ветвях; U1 и U2 — напряжение на ветвях; R1 и R2 — сопротивления веток.

По закону разветвленной цепи: I = I1 + I2

Подставляя значения токов, получим U / R = U1 / R1 + U2 / R2

Потому что при параллельном соединении U = U1 = U2 , то можем написать U / R = U / R1 + U / R2

Вынеся U в правой части равенства за скобки, получим U / R = U (1 / R1 + 1 / R2 )

Разделив сейчас обе части равенства на U , будем совсем иметь 1 / R = 1 / R1 + 1 / R2

Помня, что проводимостью именуется величина, оборотная сопротивлению , мы можем сказать, что в приобретенной формуле 1 / R — проводимость наружной цепи; 1 / R1 проводимость первой ветки; 1 / R2- проводимость 2-ой ветки.

На основании этой формулы делаем вывод: при параллельном соединении проводимость наружной цепи равна сумме проводимостей отдельных веток.

Как следует, чтоб найти эквивалентное сопротивление включенных параллельно сопротивлений, нужно найти проводимость цепи и взять величину, ей оборотную.

Из формулы также следует, что проводимость цепи больше проводимости каждой ветки, а это означает, что эквивалентное сопротивление наружной цепи меньше меньшего из включенных параллельно сопротивлений.

Рассматривая случай параллельного соединения сопротивлений, мы взяли более ординарную цепь, состоящую из 2-ух веток. Но на практике могут повстречаться случаи, когда цепь состоит из 3-х и поболее параллельных веток. Как поступать в этих случаях?

Оказывается, все приобретенные нами соотношения остаются справедливыми и для цепи, состоящей из хоть какого числа параллельно соединенных сопротивлений.

Чтоб убедиться в этом, разглядим последующий пример.

Возьмем три сопротивления R1 = 10 Ом, R2 = 20 Ом и R3 = 60 Ом и соединим их параллельно. Определим эквивалентное сопротивление цепи (рис. 5 ). R = 1 / 6 Как следует, эквивалентное сопротивление R = 6 Ом.

Таким макаром, эквивалентное сопротивление меньше меньшего из включенных параллельно в цепь сопротивлений , т. е. меньше сопротивления R1.

Поглядим сейчас, вправду ли это сопротивление является эквивалентным, т. е. таким, которое могло бы поменять включенные параллельно сопротивления в 10, 20 и 60 Ом, не изменяя при всем этом силы тока до разветвления цепи.

Допустим, что напряжение наружной цепи, а как следует, и напряжение на сопротивлениях R1, R2, R3 равно 12 В. Тогда сила токов в ветвях будет: I1 = U/R1 = 12 / 10 = 1 ,2 А I 2 = U/R 2 = 12 / 20 = 1 ,6 А I 3 = U/R1 = 12 / 60 = 0,2 А

Общий ток в цепи получим, пользуясь формулой I = I1 + I2 + I3 =1,2 + 0,6 + 0,2 = 2 А.

Проверим по формуле закона Ома, получится ли в цепи ток силой 2 А, если заместо 3-х параллельно включенных узнаваемых нам сопротивлений включено одно эквивалентное им сопротивление 6 Ом.

I = U / R = 12 / 6 = 2 А

Как лицезреем, отысканное нами сопротивление R = 6 Ом вправду является для данной цепи эквивалентным.

В этом можно убедиться и на измерительных устройствах, если собрать схему с взятыми нами сопротивлениями, измерить ток во наружной цепи (до разветвления), потом поменять параллельно включенные сопротивления одним сопротивлением 6 Ом и опять измерить ток. Показания амперметра и в том и в другом случае будут приблизительно схожими.

На практике могут повстречаться также параллельные соединения, для которых высчитать эквивалентное сопротивление можно проще, т. е. не определяя за ранее проводимостей, сходу отыскать сопротивление.

К примеру, если соединены параллельно два сопротивления R1 и R2 , то формулу 1 / R = 1 / R1 + 1 / R2 можно конвертировать так: 1/R = (R2 + R1) / R1 R2 и, решая равенство относительно R, получить R = R1 х R2 / (R1 + R2 ), т. е. при параллельном соединении 2-ух сопротивлений эквивалентное сопротивление цепи равно произведению включенных параллельно сопротивлений, деленному на их сумму.

Содержание:

Все известные виды проводников обладают определенными свойствами, в том числе и электрическим сопротивлением. Это качество нашло свое применение в резисторах, представляющих собой элементы цепи с точно установленным сопротивлением. Они позволяют выполнять регулировку тока и напряжения с высокой точностью в схемах. Все подобные сопротивления имеют свои индивидуальные качества. Например, мощность при паралл ельном и последовательном соединении резисторов будет различной. Поэтому на практике очень часто используются различные методики расчетов, благодаря которым возможно получение точных результатов.

Свойства и технические характеристики резисторов

Как уже отмечалось, резисторы в электрических цепях и схемах выполняют регулировочную функцию. С этой целью используется закон Ома, выраженный формулой: I = U/R. Таким образом, с уменьшением сопротивления происходит заметное возрастание тока. И, наоборот, чем выше сопротивление, тем меньше ток. Благодаря этому свойству, резисторы нашли широкое применение в электротехнике. На этой основе создаются делители тока, использующиеся в конструкциях электротехнических устройств.

Помимо функции регулировки тока, резисторы применяются в схемах делителей напряжения. В этом случае закон Ома будет выглядеть несколько иначе: U = I x R. Это означает, что с ростом сопротивления происходит увеличение напряжения. На этом принципе строится вся работа устройств, предназначенных для деления напряжения. Для делителей тока используется паралл ельное соединение резисторов, а для — последовательное.

На схемах резисторы отображаются в виде прямоугольника, размером 10х4 мм. Для обозначения применяется символ R, который может быть дополнен значением мощности данного элемента. При мощности свыше 2 Вт, обозначение выполняется с помощью римских цифр. Соответствующая надпись наносится на схеме возле значка резистора. Мощность также входит в состав , нанесенной на корпус элемента. Единицами измерения сопротивления служат ом (1 Ом), килоом (1000 Ом) и мегаом (1000000 Ом). Ассортимент резисторов находится в пределах от долей ома до нескольких сотен мегаом. Современные технологии позволяют изготавливать данные элементы с довольно точными значениями сопротивления.

Важным параметром резистора считается отклонение сопротивления. Его измерение осуществляется в процентах от номинала. Стандартный ряд отклонений представляет собой значения в виде: + 20, + 10, + 5, + 2, + 1% и так далее до величины + 0,001%.

Большое значение имеет мощность резистора. По каждому из них во время работы проходит электрический ток, вызывающий нагрев. Если допустимое значение рассеиваемой мощности превысит норму, это приведет к выходу из строя резистора. Следует учитывать, что в процессе нагревания происходит изменение сопротивления элемента. Поэтому если устройства работают в широких диапазонах температур, применяется специальная величина, именуемая температурным коэффициентом сопротивления.

Для соединения резисторов в схемах используются три разных способа подключения — паралл ельное, последовательное и смешанное. Каждый способ обладает индивидуальными качествами, что позволяет применять данные элементы в самых разных целях.

Мощность при последовательном соединение

При соединение резисторов последовательно электрический ток по очереди проходит через каждое сопротивление. Значение тока в любой точке цепи будет одинаковым. Данный факт определяется с помощью закона Ома. Если сложить все сопротивления, приведенные на схеме, то получится следующий результат: R = 200+100+51+39 = 390 Ом.

Учитывая напряжение в цепи, равное 100 В, сила тока будет составлять I = U/R = 100/390 = 0,256 A.На основании полученных данных можно рассчитать мощность резисторов при последовательном соединении по следующей формуле: P = I 2 x R = 0,256 2 x 390 = 25,55 Вт.

  • P 1 = I 2 x R 1 = 0,256 2 x 200 = 13,11 Вт;
  • P 2 = I 2 x R 2 = 0,256 2 x 100 = 6,55 Вт;
  • P 3 = I 2 x R 3 = 0,256 2 x 51 = 3,34 Вт;
  • P 4 = I 2 x R 4 = 0,256 2 x 39 = 2,55 Вт.

Если сложить полученные мощность, то полная Р составит: Р = 13,11+6,55+3,34+2,55 = 25,55 Вт.

Мощность при паралл ельном соединение

При паралл ельном подключении все начала резисторов соединяются с одним узлом схемы, а концы — с другим. В этом случае происходит разветвление тока, и он начинает протекать по каждому элементу. В соответствии с законом Ома, сила тока будет обратно пропорциональна всем подключенным сопротивлениям, а значение напряжения на всех резисторах будет одним и тем же.

Прежде чем вычислять силу тока, необходимо выполнить расчет полной проводимости всех резисторов, применяя следующую формулу:

  • 1/R = 1/R 1 +1/R 2 +1/R 3 +1/R 4 = 1/200+1/100+1/51+1/39 = 0,005+0,01+0,0196+0,0256 = 0,06024 1/Ом.
  • Поскольку сопротивление является величиной, обратно пропорциональной проводимости, его значение составит: R = 1/0,06024 = 16,6 Ом.
  • Используя значение напряжения в 100 В, по закону Ома рассчитывается сила тока: I = U/R = 100 x 0,06024 = 6,024 A.
  • Зная силу тока, мощность резисторов, соединенных паралл ельно, определяется следующим образом: P = I 2 x R = 6,024 2 x 16,6 = 602,3 Вт.
  • Расчет силы тока для каждого резистора выполняется по формулам: I 1 = U/R 1 = 100/200 = 0,5A; I 2 = U/R 2 = 100/100 = 1A; I 3 = U/R 3 = 100/51 = 1,96A; I 4 = U/R 4 = 100/39 = 2,56A. На примере этих сопротивлений прослеживается закономерность, что с уменьшением сопротивления, сила тока увеличивается.

Существует еще одна формула, позволяющая рассчитать мощность при паралл ельном подключении резисторов: P 1 = U 2 /R 1 = 100 2 /200 = 50 Вт; P 2 = U 2 /R 2 = 100 2 /100 = 100 Вт; P 3 = U 2 /R 3 = 100 2 /51 = 195,9 Вт; P 4 = U 2 /R 4 = 100 2 /39 = 256,4 Вт. Сложив мощности отдельных резисторов, получится их общая мощность: Р = Р 1 +Р 2 +Р 3 +Р 4 = 50+100+195,9+256,4 = 602,3 Вт.

Таким образом, мощность при последовательном и паралл ельном соединении резисторов определяется разными способами, с помощью которых можно получить максимально точные результаты.

Формула расчета сопротивления при параллельном соединении резисторов

Электрическое сопротивление характеризует свойство проводника препятствовать прохождению через него электрического тока. У каждого материала есть свое удельное сопротивление. Это табличная величина, и условно она считается постоянной.

Условно, потому что во многом эта характеристика зависит от внешних условий, например температуры. Сопротивление же какого-либо конкретного элемента (мы будем говорить о резисторах) складывается из многих факторов, например, из геометрических параметров, а когда речь идет о цепи переменного тока, то в расчеты включают также индуктивное и емкостное сопротивление, но об этом мы расскажем позже. Пока же — немного теории.

Закон Ома

В 1826 году немецкий физик Георг Ом на основе своих опытов вывел закон, согласно которому сила тока на участке цепи прямо пропорциональна напряжению, которое к нему приложено, и обратно пропорциональна сопротивлению участка. Из школьного курса мы знаем этот закон:

I=U/R

Позже он был сформулирован и для полной цепи:

I=ε/(R+r)

Где ε — ЭДС источника, R — сопротивление цепи, а r — сопротивление источника.

Мощность прибора

Электрический заряд при своем перемещении совершает работу. Может быть, это незаметно глазу, но вот пощупать результат этой работы можно: электроприборы у нас греются, а иногда нагрев — это цель, а не побочное явление. Не верите — ну, электроплитки, ТЭНы, утюги как раз это свойство и эксплуатируют. Правда, руками это проверять не советую.

Мощностью у нас называют работу, совершенную за единицу времени. Попробуем вычислить мощность электроприбора, включенного в цепь.2/R

Последовательное и параллельное соединение

В реальной жизни мы редко имеем дело с одним проводником и одним источником. Достаточно взглянуть в любую принципиальную электрическую схему, например, такую простенькую:

(это схема микроволновки «Электроника»)

можно увидеть, что элементы в схеме соединены по-разному, но мы покажем вам базовые закономерности, которые работают в цепях.

Правила Кирхгофа

Если взять замкнутую электрическую цепь, по которой течет заряд, то можно определенно сказать: он никуда не денется. Сумма всех зарядов, которые текут в одной цепи, всегда одинакова. Это называется законом сохранения заряда, частным случаем общего закона сохранения (как говорится, если в одном месте что-то убудет, в другом непременно прибудет).

Отсюда мы и выводим тот факт, что в каждом узле цепи сумма токов равна нулю. То есть, если ток «приходит» в точку по ветке и «уходит» по двум — значит, первый равен сумме второго и третьего.

На этой картинке мы видим, что I1+I4=I2+I3

Это называется первым правилом Кирхгофа.

Если наша цепь не будет содержать узлов, значит, ток в ней будет величиной постоянной, а элементы, один за другим поставленные в цепь, будут давать падение напряжения. При этом общее напряжение в цепи останется тем же. Отсюда вытекает второе правило Кирхгофа: сумма напряжений на участках цепи будет равна ЭДС источников тока, входящий в эту цепь. Если у нас источник один, то будет верно равенство:

ε=U1+U2+U3+…+Un

Сумма падений напряжения будет, таким образом, нулевой.

В ситуациях, когда мы имеем дело с переменным током, падение будет наблюдаться на участках с конденсаторами и катушками — в цепях переменного тока у них появляется сопротивление (об этом позже).

Теперь, когда мы познакомились с теоретической частью, можем перейти к более приближенному к суровой реальности вопросу, а именно — расчету последовательного и параллельного соединения резисторов.2/R

Исходя из вышеперечисленных закономерностей, вы сможете рассчитывать самые причудливые соединения резисторов, можете попрактиковаться, взяв в библиотеке задачник.

Типы резисторов

Как уже было сказано ранее, элемент, который ставится в цепь для нагрузки, называется резистором. Ставят его для разных целей, главным образом для того, чтобы изменить тот или иной параметр на участке цепи. Например, понизить напряжение или силу тока, чтобы деталь, стоящая за резистором, не сгорела.

Предприятиями выпускается большой ассортимент таких изделий, и их можно по-разному классифицировать. Номинально резистор имеет то сопротивление, которое указано на нем, а по факту оно может зависеть от напряжения в сети (нелинейность), иметь разброс параметра (иногда до 20% доходит). По применяемой технологии резисторы можно разделить на:

  1. проволочные;
  2. композитные;
  3. металлофольговые;
  4. угольные;
  5. интегральные.

Фактическое сопротивление такого элемента может зависеть от температуры окружающей среды и даже от частоты, если мы имеем дело с переменным током. Дело в том, что часть ассортимента резисторов выполнены по проволочной технологии, то есть фактически они представляют собой мини-катушку. При малых частотах (50 Гц) это в расчет не берется, а вот на высоких (мегагерцы) паразитная индуктивность и индуктивное сопротивление может сказаться на работе схемы. Поэтому при выборе резистора для работы с высокочастотными схемами внимательно смотрите. по какой технологии он сделан. Отдайте предпочтение тонкослойным и композиционным изделиям.

Помимо этого, большое распространение получили переменные резисторы, значение сопротивления которых можно регулировать. Делается это чаще всего отверткой. Необходимость в таких изделиях продиктована разбросом параметров у обычных резисторов, а подстроечный вариант позволяет регулировать сопротивление.

Все вышесказанное актуально для цепей постоянного тока и переменного при невысоких частотах, и все это — при нормальных условиях внешней среды. Расчеты цепей при нарушении этих условий нуждаются в дополнительной корректировке: это связано с ограниченностью действия закона Ома. С чем связаны ограничения? Вот несколько примеров:

  1. при сверхнизких температурах многие проводники проявляют такое интересное явление, как сверхпроводимость;
  2. также сопротивление может разниться при нагревании;
  3. неприменим закон Ома для описания электрического тока в газах;
  4. наконец, обычный резистор можно просто пробить высоким напряжением.

Все это прекрасно работает. Не верите — можете поэкспериментировать у себя дома или провести замеры тестером. Например, изучить елочную гирлянду или показания счетчиков при включенных электроприборах (напомню, что в гирлянде лампочки соединены последовательно, а розетки в доме — параллельно). Удачи!

Как я и обещал в статье про переменные резисторы (ссылка), сегодня речь пойдет о возможных способах соединения резисторов, в частности о последовательном соединении и о параллельном.

Последовательное соединение резисторов.

Давайте начнем с рассмотрения цепей, элементы которой соединены последовательно. И хоть мы и будем рассматривать только резисторы в качестве элементов цепи в данной статье, но правила, касающиеся напряжений и токов при разных соединениях будут справедливы и для других элементов. Итак, первая цепь, которую мы будем разбирать выглядит следующим образом:

Здесь у нас классический случай последовательного соединения – два последовательно включенных резистора. Но не будем забегать вперед и рассчитывать общее сопротивление цепи, а для начала рассмотрим все напряжения и токи. Итак, первое правило заключается в том, что протекающие по всем проводникам токи при последовательном соединении равны между собой:

А для определения общего напряжения при последовательном соединении, напряжения на отдельных элементах необходимо просуммировать:

В то же время, по закону Ома для напряжений, сопротивлений и токов в данной цепи справедливы следующие соотношения:

Тогда для вычисления общего напряжения можно будет использовать следующее выражение:

Но для общего напряжение также справедлив закон Ома:

Здесь – это общее сопротивление цепи, которое исходя из двух формул для общего напряжения равно:

Таким образом, при последовательном соединении резисторов общее сопротивление цепи будет равно сумме сопротивлений всех проводников.

Например для следующей цепи:

Общее сопротивление будет равно:

Количество элементов значения не имеет, правило, по которому мы определяем общее сопротивление будем работать в любом случае 🙂 А если при последовательном соединении все сопротивления равны (), то общее сопротивление цепи составит:

в данной формуле равно количеству элементов цепи.

С последовательным соединением резисторов мы разобрались, давайте перейдем к параллельному.

Параллельное соединение резисторов.

При параллельном соединении напряжения на проводниках равны:

А для токов справедливо следующее выражение:

То есть общий ток разветвляется на две составляющие, а его значение равно сумме всех составляющих. По закону Ома:

Подставим эти выражения в формулу общего тока:

А по закону Ома ток:

Приравниваем эти выражения и получаем формулу для общего сопротивления цепи:

Данную формулу можно записать и несколько иначе:

Таким образом, при параллельном соединении проводников величина, обратная общему сопротивлению цепи, равна сумме величин, обратных сопротивлениям параллельно включенных проводников.

Аналогичная ситуация будет наблюдаться и при большем количестве проводников, соединенных параллельно:

Смешанное соединение резисторов.

Помимо параллельного и последовательного соединений резисторов существует еще смешанное соединение. Из названия уже понятно, что при таком соединении в цепи присутствуют резисторы, соединенные как параллельно, так и последовательно. Вот пример такой цепи:

Давайте рассчитаем общее сопротивление цепи. Начнем с резисторов и – они соединены параллельно. Мы можем рассчитать общее сопротивление для этих резисторов и заменить их в схеме одним единственным резистором :

Теперь у нас образовались две группы последовательно соединенных резисторов:

Заменим эти две группы двумя резисторами, сопротивление которых равно:

Как видите, схема стала уже совсем простой ) Заменим группу параллельно соединенных резисторов и одним резистором :

И в итоге у нас на схеме осталось только два резистора соединенных последовательно:

Общее сопротивление цепи получилось равным:

Таким вот образом достаточно большая схема свелась к простейшему последовательному соединению двух резисторов 😉

Тут стоит отметить, что некоторые схемы невозможно так просто преобразовать и определить общее сопротивление – для таких схем нужно использовать правила Кирхгофа, о которых мы обязательно поговорим в будущих статьях. А сегодняшняя статья на этом подошла к концу, до скорых встреч на нашем сайте!

Проверим справедливость показанных здесь формул на простом эксперименте.

Возьмём два резистора МЛТ-2 на 3 и 47 Ом и соединим их последовательно. Затем измерим общее сопротивление получившейся цепи цифровым мультиметром. Как видим оно равно сумме сопротивлений резисторов, входящих в эту цепочку.


Замер общего сопротивления при последовательном соединении

Теперь соединим наши резисторы параллельно и замерим их общее сопротивление.


Измерение сопротивления при параллельном соединении

Как видим, результирующее сопротивление (2,9 Ом) меньше самого меньшего (3 Ом), входящего в цепочку. Отсюда вытекает ещё одно известное правило, которое можно применять на практике:

При параллельном соединении резисторов общее сопротивление цепи будет меньше наименьшего сопротивления, входящего в эту цепь.

Что ещё нужно учитывать при соединении резисторов?

Во-первых, обязательно учитывается их номинальная мощность. Например, нам нужно подобрать замену резистору на 100 Ом и мощностью 1 Вт . Возьмём два резистора по 50 Ом каждый и соединим их последовательно. На какую мощность рассеяния должны быть рассчитаны эти два резистора?

Поскольку через последовательно соединённые резисторы течёт один и тот же постоянный ток (допустим 0,1 А ), а сопротивление каждого из них равно 50 Ом , тогда мощность рассеивания каждого из них должна быть не менее 0,5 Вт . В результате на каждом из них выделится по 0,5 Вт мощности. В сумме это и будет тот самый 1 Вт .

Данный пример достаточно грубоват. Поэтому, если есть сомнения, стоит брать резисторы с запасом по мощности.

Подробнее о мощности рассеивания резистора читайте .

Во-вторых, при соединении стоит использовать однотипные резисторы, например, серии МЛТ. Конечно, нет ничего плохого в том, чтобы брать разные. Это лишь рекомендация.

Параллельное соединение резисторов — одно из двух видов электрических соединений, когда оба вывода одного резистора соединены с соответствующими выводами другого резистора или резисторов. Зачастую или параллельно для того, чтобы создать более сложные электронные схемы.

Схема параллельного соединения показан на рисунке ниже. При параллельном соединении резисторов, напряжение на всех резисторах будет одинаковым, а протекающий через них ток будет пропорционален их сопротивлению:

Формула параллельного соединения резисторов

Общее сопротивление нескольких резисторов соединенных параллельно определяется по следующей формуле:

Ток, протекающий через отдельно взятый резистор, согласно , можно найти по формуле:

Параллельное соединение резисторов — расчет

Пример №1

При разработке устройства, возникла необходимость установить резистор с сопротивлением 8 Ом. Если мы просмотрим весь номинальный ряд стандартных значений резисторов, то мы увидим, что резистора с сопротивлением в 8 Ом в нем нет.

Выходом из данной ситуации будет использование двух параллельно соединенных резисторов. Эквивалентное значение сопротивления для двух резисторов соединенных параллельно рассчитывается следующим образом:

Данное уравнение показывает, что если R1 равен R2, то сопротивление R составляет половину сопротивления одного из двух резисторов. При R = 8 Ом, R1 и R2 должны, следовательно, иметь значение 2 × 8 = 16 Ом.
Теперь проведем проверку, рассчитав общее сопротивление двух резисторов:

Таким образом, мы получили необходимое сопротивление 8 Ом, соединив параллельно два резистора по 16 Ом.

Пример расчета №2

Найти общее сопротивление R из трех параллельно соединенных резисторов:

Общее сопротивление R рассчитывается по формуле:

Этот метод расчета может быть использованы для расчета любого количества отдельных сопротивлений соединенных параллельно.

Один важный момент, который необходимо запомнить при расчете параллельно соединенных резисторов – это то, что общее сопротивление всегда будет меньше, чем значение наименьшего сопротивления в этой комбинации.

Как рассчитать сложные схемы соединения резисторов

Более сложные соединения резисторов могут быть рассчитаны путем систематической группировки резисторов. На рисунке ниже необходимо посчитать общее сопротивление цепи, состоящей из трех резисторов:


Для простоты расчета, сначала сгруппируем резисторы по параллельному и последовательному типу соединения.
Резисторы R2 и R3 соединены последовательно (группа 2). Они в свою очередь соединены параллельно с резистором R1 (группа 1).

Последовательное соединение резисторов группы 2 вычисляется как сумма сопротивлений R2 и R3:

В результате мы упрощаем схему в виде двух параллельных резисторов. Теперь общее сопротивление всей схемы можно посчитать следующим образом:

Расчет более сложных соединений резисторов можно выполнить используя законы Кирхгофа.

Ток, протекающий в цепи параллельно соединенных резисторах

Общий ток I протекающий в цепи параллельных резисторов равняется сумме отдельных токов, протекающих во всех параллельных ветвях, причем ток в отдельно взятой ветви не обязательно должен быть равен току в соседних ветвях.

Несмотря на параллельное соединение, к каждому резистору приложено одно и то же напряжение. А поскольку величина сопротивлений в параллельной цепи может быть разной, то и величина протекающего тока через каждый резистор тоже будет отличаться (по определению закона Ома).

Рассмотрим это на примере двух параллельно соединенных резисторов. Ток, который течет через каждый из резисторов (I1 и I2) будет отличаться друг от друга поскольку сопротивления резисторов R1 и R2 не равны.
Однако мы знаем, что ток, который поступает в цепь в точке «А» должен выйти из цепи в точке «B» .

Первое правило Кирхгофа гласит: «Общий ток, выходящий из цепи равен току входящий в цепь».

Таким образом, протекающий общий ток в цепи можно определить как:

Затем с помощью закона Ома можно вычислить ток, который протекает через каждый резистор:

Ток, протекающий в R1 = U ÷ R1 = 12 ÷ 22 кОм = 0,545 мА

Ток, протекающий в R 2 = U ÷ R2 = 12 ÷ 47 кОм = 0,255 мА

Таким образом, общий ток будет равен:

I = 0,545 мА + 0,255 мА = 0,8 мА

Это также можно проверить, используя закон Ома:

I = U ÷ R = 12 В ÷ 15 кОм = 0,8 мА (то же самое)

где 15кОм — это общее сопротивление двух параллельно соединенных резисторов (22 кОм и 47 кОм)

И в завершении хочется отметить, что большинство современных резисторов маркируются цветными полосками и назначение ее можно узнать .

Параллельное соединение резисторов — онлайн калькулятор

Чтобы быстро вычислить общее сопротивление двух и более резисторов, соединенных параллельно, вы можете воспользоваться следующим онлайн калькулятором:

Подведем итог

Когда два или более резистора соединены так, что оба вывода одного резистора соединены с соответствующими выводами другого резистора или резисторов, то говорят, что они соединены между собой параллельно. Напряжение на каждом резисторе внутри параллельной комбинации одинаковое, но токи, протекающие через них, могут отличаться друг от друга, в зависимости от величины сопротивлений каждого резистора.

Эквивалентное или полное сопротивление параллельной комбинации всегда будет меньше минимального сопротивления резистора входящего в параллельное соединение.

На практике нередко встречается задача нахождения сопротивления проводников и резисторов при различных способах соединения. В статье рассмотрено, как рассчитывается сопротивление при и некоторые другие технические вопросы.

Сопротивление проводника

Все проводники имеют свойство препятствовать течению электрического тока, его принято называть электрическим сопротивлением R, оно измеряется в омах. Это основное свойство проводниковых материалов.

Для ведения электротехнических расчётов применяется удельное сопротивление – ρ Ом·м/мм 2 . Все металлы – хорошие проводники, наибольшее применение получили медь и алюминий, гораздо реже применяется железо. Лучший проводник – серебро, оно применяется в электротехнической и электронной промышленности. Широко распространены сплавы с высоким значением сопротивления.

При расчёте сопротивления используется известная из школьного курса физики формула:

R = ρ · l/S, S – площадь сечения; l – длина.

Если взять два проводника, то их сопротивление при параллельном соединении станет меньше из-за увеличения общего сечения.

и нагрев проводника

Для практических расчётов режимов работы проводников применяется понятие плотности тока – δ А/мм 2 , она вычисляется по формуле:

δ = I/S, I – ток, S – сечение.

Ток, проходя по проводнику, нагревает его. Чем больше δ, тем сильнее нагревается проводник. Для проводов и кабелей разработаны нормы допустимой плотности, которые приводятся в Для проводников нагревательных устройств существуют свои нормы плотности тока.

Если плотность δ выше допустимой, может произойти разрушение проводника, например, при перегреве кабеля у него разрушается изоляция.

Правилами регламентируется производить расчёт проводников на нагрев.

Способы соединения проводников

Любой проводник гораздо удобнее изображать на схемах как электрическое сопротивление R, тогда их легко читать и анализировать. Существует всего три способа соединения сопротивлений. Первый способ самый простой – последовательное соединение.

На фото видно, что полное сопротивление равно: R = R 1 + R 2 + R 3 .

Второй способ более сложный – параллельное соединение. Расчёт сопротивления при параллельном соединении выполняется поэтапно. Рассчитывается полная проводимость G = 1/R, а затем полное сопротивление R = 1/G.

Можно поступить и по-другому, прежде рассчитать общее сопротивление при R1 и R2, после этого повторить операцию и найти R.

Третий способ соединения наиболее сложный – смешанное соединение, то есть присутствуют все рассмотренные варианты. Схема приведена на фото.

Для расчёта этой схемы её следует упростить, для этого заменяют резисторы R2 и R3 одним R2,3. Получается несложная схема.

R2,3,4 = R2,3 · R4/(R2,3 + R4).

Схема становится ещё проще, в ней остаются резисторы, имеющие последовательное соединение. В более сложных ситуациях используется этот же метод преобразования.

Виды проводников

В электронной технике, при производстве проводники представляют собою тонкие полоски медной фольги. Ввиду малой длины сопротивление у них незначительно, им во многих случаях можно пренебречь. Для этих проводников сопротивление при параллельном соединении уменьшается вследствие увеличения сечения.

Большой раздел проводников представляют обмоточные провода. Они выпускаются разных диаметров – от 0,02 до 5,6 миллиметра. Для мощных трансформаторов и электродвигателей выпускаются медные шинки прямоугольного сечения. Иногда при ремонте заменяют провод большого диаметра на несколько параллельно соединённых меньшего размера.

Особый раздел проводников представляют провода и кабели, промышленность предоставляет широчайший выбор марок для самых различных нужд. Нередко приходится заменять один кабель на несколько, меньшего сечения. Причины этого бывают самые различные, например, кабель сечением 240 мм 2 очень трудно прокладывать по трассе с крутыми изгибами. Его заменяют на 2×120 мм 2 , и проблема решена.

Расчёт проводов на нагрев

Проводник нагревается протекающим током, если его температура превысит допустимую, наступает разрушение изоляции. ПУЭ предусматривает расчёт проводников на нагрев, исходными данными для него являются сила тока и условия внешней среды, в которой проложен проводник. По этим данным из таблиц в ПУЭ выбирается рекомендуемое проводника или кабеля).

На практике встречаются ситуации, когда нагрузка на действующий кабель сильно возросла. Существует два выхода ‒ заменить кабель на другой, это бывает дорого, или параллельно ему проложить ещё один, чтобы разгрузить основной кабель. В этом случае сопротивление проводника при параллельном соединении уменьшается, следовательно падает выделение тепла.

Чтобы правильно выбрать сечение второго кабеля, пользуются таблицами ПУЭ, важно при этом не ошибиться с определением его рабочего тока. В этой ситуации охлаждение кабелей будет даже лучше, чем у одного. Рекомендуется рассчитать сопротивление при параллельном соединении двух кабелей, чтобы точнее определить их тепловыделение.

Расчёт проводников на потерю напряжения

При расположении потребителя R н на большом расстоянии L от источника энергии U 1 возникает довольно большое на проводах линии. К потребителю R н поступает напряжение U 2 значительно ниже начального U 1 . Практически в качестве нагрузки выступает различное электрооборудование, подключаемое к линии параллельно.

Для решения проблемы производят расчет сопротивления при параллельном соединении всего оборудования, так находится сопротивление нагрузки R н. Далее следует определить сопротивление проводов линии.

Здесь S – сечение провода линии, мм 2 .

Каждый в этой жизни сталкивался с резисторами. Люди с гуманитарными профессиями, как и все, изучали в школе на уроках физики проводники электрического тока и закон Ома.

С резисторами также имеют дело студенты технических университетов и инженеры различных производственных предприятий. Перед всеми этими людьми, так или иначе, вставала задача расчёта электрической цепи при различных видах соединения резисторов. В данной статье речь пойдёт о расчёте физических параметров, характеризующих цепь.

Виды соединений

Резистор – пассивный элемент , присутствующий в каждой электрической цепи. Он предназначен для того, чтобы сопротивляться электрическому току. Существует два вида резисторов:

Зачем же спаивать проводники друг с другом? Например, если для какой-то электрической цепи нужно определённое сопротивление. А среди номинальных показателей нужного нет. В таком случае необходимо подобрать элементы схемы с определёнными значениями сопротивления и соединить их. В зависимости от вида соединения и сопротивлений пассивных элементов мы получим какое-то определённое сопротивление цепи. Оно называется эквивалентным. Его значение зависит от вида спайки проводников. Существует три вида соединения проводников:

Значение эквивалентного сопротивления в цепи считается достаточно легко. Однако, если резисторов в схеме очень много, то лучше воспользоваться специальным калькулятором, который считает это значение. При ведении расчёта вручную, чтобы не допускать ошибок, необходимо проверять, ту ли формулу вы взяли.

Последовательное соединение проводников

В последовательной спайке резисторы идут как бы друг за другом. Значение эквивалентного сопротивления цепи равно сумме сопротивлений всех резисторов. Особенность схем с такой спайкой заключается в том, что значение тока постоянно . Согласно закону Ома, напряжение в цепи равно произведению тока и сопротивления. Так как ток постоянен, то для вычисления напряжения на каждом резисторе, достаточно перемножить значения. После этого необходимо сложить напряжения всех резисторов, и тогда мы получим значение напряжения во всей цепи.

Расчёт очень простой. Так как с ним имеют дело в основном инженеры-разработчики, то для них не составит труда сосчитать всё вручную. Но если резисторов очень много, то проще воспользоваться специальным калькулятором.

Примером последовательного соединения проводников в быту является ёлочная гирлянда.

Параллельное соединение резисторов

При параллельном соединении проводников эквивалентное сопротивление в цепи считается по-другому. Немного сложнее, чем при последовательном.

Его значение в таких цепях равняется произведению сопротивлений всех резисторов, делённому на их сумму. А также есть и другие варианты этой формулы. Параллельное соединение резисторов всегда снижает эквивалентное сопротивление цепи. То есть, его значение всегда будет меньше, чем наибольшее значение какого-то из проводников.

В таких схемах значение напряжения постоянно . То есть значение напряжения во всей цепи равно значениям напряжений каждого из проводников. Оно задаётся источником напряжения.

Сила тока в цепи равна сумме всех токов, протекающих через все проводники. Значение силы тока, протекающего через проводник. равно отношению напряжения источника к сопротивлению этого проводника.

Примеры параллельного соединения проводников:

  1. Освещение.
  2. Розетки в квартире.
  3. Производственное оборудование.

Для расчёта схем с параллельным соединением проводников лучше пользоваться специальным калькулятором. Если в схеме много резисторов, спаянных параллельно, то гораздо быстрее вы посчитаете эквивалентное сопротивление с помощью этого калькулятора.

Смешанное соединение проводников

Этот вид соединения состоит из каскадов резисторов . Например, у нас есть каскад из 10 проводников, соединённых последовательно, и после него идёт каскад из 10 проводников, соединённых параллельно. Эквивалентное сопротивление этой схемы будет равно сумме эквивалентных сопротивлений этих каскадов. То есть, по сути, здесь последовательное соединение двух каскадов проводников.

Многие инженеры занимаются оптимизацией различных схем. Её целью является уменьшение количества элементов в схеме за счёт подбора других, с подходящими значениями сопротивлений. Сложные схемы разбиваются на несколько небольших каскадов, ведь так гораздо проще вести расчёты.

Сейчас, в двадцать первом веке, инженерам стало гораздо проще работать. Ведь несколько десятилетий назад все расчёты производились вручную. А сейчас программисты разработали специальный калькулятор для расчёта эквивалентного сопротивления цепи. В нём запрограммированы формулы, по которым ведутся расчёты.

В этом калькуляторе можно выбрать вид соединения, и потом ввести в специальные поля значения сопротивлений. Через несколько секунд вы уже увидите это значение.

Последовательное и параллельное соединение резисторов.

Как я и обещал в статье про переменные резисторы (ссылка), сегодня речь пойдет о возможных способах соединения, в частности о последовательном соединении резисторов и о параллельном.

Последовательное соединение резисторов.

Давайте начнем с рассмотрения цепей, элементы которой соединены последовательно. И хоть мы и будем рассматривать только резисторы в качестве элементов цепи в данной статье, но правила, касающиеся напряжений и токов при разных соединениях будут справедливы и для других элементов. Итак, первая цепь, которую мы будем разбирать выглядит следующим образом:

Здесь у нас классический случай последовательного соединения – два последовательно включенных резистора. Но не будем забегать вперед и рассчитывать общее сопротивление цепи, а для начала рассмотрим все напряжения и токи. Итак, первое правило заключается в том, что протекающие по всем проводникам токи при последовательном соединении равны между собой:

I = I_1 = I_2

А для определения общего напряжения при последовательном соединении, напряжения на отдельных элементах необходимо просуммировать:

U = U_1 + U_2

В то же время, по закону Ома для напряжений, сопротивлений и токов в данной цепи справедливы следующие соотношения:

U_1 = I_1R_1 = IR_1

U_2 = I_2R_2 = IR_2

Тогда для вычисления общего напряжения можно будет использовать следующее выражение:

U = U_1 + U_2 = IR_2 + IR_2 = I(R_1 + R_2)

Но для общего напряжение также справедлив закон Ома:

U = IR_0

Здесь R_0 – это общее сопротивление цепи, которое исходя из двух формул для общего напряжения равно:

R_0 = R_1 + R_2

Таким образом, при последовательном соединении резисторов общее сопротивление цепи будет равно сумме сопротивлений всех проводников.

Например для следующей цепи:

Общее сопротивление будет равно:

R_0 = R_1 + R_2 + R_3 + R_4 + R_5 + R_6 + R_7 + R_8 + R_9 + R_{10}

Количество элементов значения не имеет, правило, по которому мы определяем общее сопротивление будем работать в любом случае 🙂 А если при последовательном  соединении все сопротивления равны (R_1 = R_2 = … = R), то общее сопротивление цепи составит:

R_0 = nR

В данной формуле n равно количеству элементов цепи. С последовательным соединением резисторов мы разобрались, давайте перейдем к параллельному.

Параллельное соединение резисторов.

При параллельном соединении напряжения на проводниках равны:

U_1 = U_2 = U

А для токов справедливо следующее выражение:

I = I_1 + I_2

То есть общий ток разветвляется на две составляющие, а его значение равно сумме всех составляющих. По закону Ома:

I_1 = \frac{U_1}{R_1} = \frac{U}{R_1}

I_2 = \frac{U_2}{R_2} = \frac{U}{R_2}

Подставим эти выражения в формулу общего тока:

I = \frac{U}{R_1} + \frac{U}{R_2} = U\medspace (\frac{1}{R1} + \frac{1}{R2})

А по закону Ома ток:

I = \frac{U}{R_0}

Приравниваем эти выражения и получаем формулу для общего сопротивления цепи:

\frac{1}{R_0} = \frac{1}{R_1} + \frac{1}{R_2}

Данную формулу можно записать и несколько иначе:

R_0 = \frac{R_1R_2}{R_1 + R_2}

Таким образом, при параллельном соединении проводников величина, обратная общему сопротивлению цепи, равна сумме величин, обратных сопротивлениям параллельно включенных проводников.

Аналогичная ситуация будет наблюдаться и при большем количестве проводников, соединенных параллельно:

\frac{1}{R_0} = \frac{1}{R_1} + \frac{1}{R_2} + \frac{1}{R_3} + \frac{1}{R_4} + \frac{1}{R_5} + \frac{1}{R_6}

Смешанное соединение резисторов.

Помимо параллельного и последовательного соединений резисторов существует еще смешанное соединение. Из названия уже понятно, что при таком соединении в цепи присутствуют резисторы, соединенные как параллельно, так и последовательно. Вот пример такой цепи:

Давайте рассчитаем общее сопротивление цепи. Начнем с резисторов R_1 и R_2 – они соединены параллельно. Мы можем рассчитать общее сопротивление для этих резисторов и заменить их в схеме одним единственным резистором R_{1-2}:

R_{1-2} = \frac{R1\cdot R2}{R1 + R2} = 1

Теперь у нас образовались две группы последовательно соединенных резисторов:

Заменим эти две группы двумя резисторами, сопротивление которых равно:

R_{1-2-3} = R_{1-2} + R_3 = 5

R_{4-5} = R_4 + R_5 = 24

Как видите, схема стала уже совсем простой 🙂 Заменим группу параллельно соединенных резисторов R_{1-2-3} и R_{4-5}  одним резистором R_{1-2-3-4-5}:

R_{1-2-3-4-5}\enspace = \frac{R_{1-2-3}\medspace\cdot R_{4-5}}{R_{1-2-3} + R_{4-5}} = \frac{5\cdot24}{5 + 24} = 4.14

И в итоге у нас на схеме осталось только два резистора соединенных последовательно:

Общее сопротивление цепи получилось равным:

R_0 = R_{1-2-3-4-5}\medspace +\medspace R_6 = 4.14 + 10 = 14.14

Таким вот образом достаточно большая схема свелась к простейшему последовательному соединению двух резисторов!

Тут стоит отметить, что некоторые схемы невозможно так просто преобразовать и определить общее сопротивление – для таких схем нужно использовать правила Кирхгофа, о которых мы обязательно поговорим в будущих статьях. А сегодняшняя статья на этом подошла к концу, до скорых встреч на нашем сайте!

Как определить сопротивление при последовательном соединении. Последовательное и параллельное соединение проводников

Причем это могут быть не только проводники, но и конденсаторы. Здесь важно не запутаться в том, как выглядит каждое из них на схеме. А уже потом применять конкретные формулы. Их, кстати, нужно помнить наизусть.

Как различить эти два соединения?

Внимательно посмотрите на схему. Если провода представить как дорогу, то машины на ней будут играть роль резисторов. На прямой дороге без каких-либо разветвлений машины едут одна за другой, в цепочку. Так же выглядит и последовательное соединение проводников. Дорога в этом случае может иметь неограниченное количество поворотов, но ни одного перекрестка. Как бы ни виляла дорога (провода), машины (резисторы) всегда будут расположены друг за другом, по одной цепочке.

Совсем другое дело, если рассматривается параллельное соединение. Тогда резисторы можно сравнить со спортсменами на старте. Они стоят каждый на своей дорожке, но направление движения у них одинаковое, и финиш в одном месте. Так же и резисторы — у каждого из них свой провод, но все они соединены в некоторой точке.

Формулы для силы тока

О ней всегда идет речь в теме «Электричество». Параллельное и последовательное соединение по-разному влияют на величину в резисторах. Для них выведены формулы, которые можно запомнить. Но достаточно просто запомнить смысл, который в них вкладывается.

Так, ток при последовательном соединении проводников всегда одинаков. То есть в каждом из них значение силы тока не отличается. Провести аналогию можно, если сравнить провод с трубой. В ней вода течет всегда одинаково. И все препятствия на ее пути будут сметаться с одной и той же силой. Так же с силой тока. Поэтому формула общей силы тока в цепи с последовательным соединением резисторов выглядит так:

I общ = I 1 = I 2

Здесь буквой I обозначена сила тока. Это общепринятое обозначение, поэтому его нужно запомнить.

Ток при параллельном соединении уже не будет постоянной величиной. При той же аналогии с трубой получается, что вода разделится на два потока, если у основной трубы будет ответвление. То же явление наблюдается с током, когда на его пути появляется разветвление проводов. Формула общей силы тока при :

I общ = I 1 + I 2

Если разветвление составлено из проводов, которых больше двух, то в приведенной формуле на такое же количество станет больше слагаемых.

Формулы для напряжения

Когда рассматривается схема, в которой выполнено соединение проводников последовательно, то напряжение на всем участке определяется суммой этих величин на каждом конкретном резисторе. Сравнить эту ситуацию можно с тарелками. Удержать одну из них легко получится одному человеку, вторую рядом он тоже сможет взять, но уже с трудом. Держать в руках три тарелки рядом друг с другом одному человеку уже не удастся, потребуется помощь второго. И так далее. Усилия людей складываются.

Формула для общего напряжения участка цепи с последовательным соединением проводников выглядит так:

U общ = U 1 + U 2 , где U — обозначение, принятое для

Другая ситуация складывается, если рассматривается Когда тарелки ставятся друг на друга, их по-прежнему может удержать один человек. Поэтому складывать ничего не приходится. Такая же аналогия наблюдается при параллельном соединении проводников. Напряжение на каждом из них одинаковое и равно тому, которое на всех них сразу. Формула общего напряжения такая:

U общ = U 1 = U 2

Формулы для электрического сопротивления

Их уже можно не запоминать, а знать формулу закона Ома и из нее выводить нужную. Из указанного закона следует, что напряжение равно произведению силы тока и сопротивления. То есть U = I * R, где R — сопротивление.

Тогда формула, с которой нужно будет работать, зависит от того, как выполнено соединение проводников:

  • последовательно, значит, нужно равенство для напряжения — I общ * R общ = I 1 * R 1 + I 2 * R 2;
  • параллельно необходимо пользоваться формулой для силы тока — U общ / R общ = U 1 / R 1 + U 2 / R 2 .

Далее следуют простые преобразования, которые основываются на том, что в первом равенстве все силы тока имеют одинаковое значение, а во втором — напряжения равны. Значит, их можно сократить. То есть получаются такие выражения:

  1. R общ = R 1 + R 2 (для последовательного соединения проводников).
  2. 1 / R общ = 1 / R 1 + 1 / R 2 (при параллельном соединении).

При увеличении числа резисторов, которые включены в сеть, изменяется количество слагаемых в этих выражениях.

Стоит отметить, что параллельное и последовательное соединение проводников по-разному влияют на общее сопротивление. Первое из них уменьшает сопротивление участка цепи. Причем оно оказывается меньше самого маленького из использованных резисторов. При последовательном соединении все логично: значения складываются, поэтому общее число всегда будет самым большим.

Работа тока

Предыдущие три величины составляют законы параллельного соединения и последовательного расположения проводников в цепи. Поэтому их знать нужно обязательно. Про работу и мощность необходимо просто запомнить базовую формулу. Она записывается так: А = I * U * t , где А — работа тока, t — время его прохождения по проводнику.

Для того чтобы определить общую работу при последовательном соединении нужно заменить в исходном выражении напряжение. Получится равенство: А = I * (U 1 + U 2) * t, раскрыв скобки в котором получится, что работа на всем участке равна их сумме на каждом конкретном потребителе тока.

Аналогично идет рассуждение, если рассматривается схема параллельного соединения. Только заменять полагается силу тока. Но результат будет тот же: А = А 1 + А 2 .

Мощность тока

При выведении формулы для мощности (обозначение «Р») участка цепи опять нужно пользоваться одной формулой: Р = U * I. После подобных рассуждений получается, что параллельное и последовательное соединение описываются такой формулой для мощности: Р = Р 1 + Р 2 .

То есть, как бы ни были составлены схемы, общая мощность будет складываться из тех, которые задействованы в работе. Именно этим объясняется тот факт, что нельзя включать в сеть квартиры одновременно много мощных приборов. Она просто не выдержит такой нагрузки.

Как влияет соединение проводников на ремонт новогодней гирлянды?

Сразу же после того, как перегорит одна из лампочек, станет ясно, как они были соединены. При последовательном соединении не будет светиться ни одна из них. Это объясняется тем, что пришедшая в негодность лампа создает разрыв в цепи. Поэтому нужно проверить все, чтобы определить, какая перегорела, заменить ее — и гирлянда станет работать.

Если в ней используется параллельное соединение, то она не перестает работать при неисправности одной из лампочек. Ведь цепь не будет полностью разорвана, а только одна параллельная часть. Чтобы отремонтировать такую гирлянду, не нужно проверять все элементы цепи, а только те, которые не светятся.

Что происходит с цепью, если в нее включены не резисторы, а конденсаторы?

При их последовательном соединении наблюдается такая ситуация: заряды от плюсов источника питания поступают только на внешние обкладки крайних конденсаторов. Те, что находятся между ними, просто передают этот заряд по цепочке. Этим объясняется то, что на всех обкладках появляются одинаковые заряды, но имеющие разные знаки. Поэтому электрический заряд каждого конденсатора, соединенного последовательно, можно записать такой формулой:

q общ = q 1 = q 2 .

Для того чтобы определить напряжение на каждом конденсаторе, потребуется знание формулы: U = q / С. В ней С — емкость конденсатора.

Общее напряжение подчиняется тому же закону, который справедлив для резисторов. Поэтому, заменив в формуле емкости напряжение на сумму, мы получим, что общую емкость приборов нужно вычислять по формуле:

С = q / (U 1 + U 2).

Упростить эту формулу можно, перевернув дроби и заменив отношение напряжения к заряду емкостью. Получается такое равенство: 1 / С = 1 / С 1 + 1 / С 2 .

Несколько по-другому выглядит ситуация, когда соединение конденсаторов — параллельное. Тогда общий заряд определяется суммой всех зарядов, которые накапливаются на обкладках всех приборов. А значение напряжения по-прежнему определяется по общим законам. Поэтому формула для общей емкости параллельно соединенных конденсаторов выглядит так:

С = (q 1 + q 2) / U.

То есть эта величина считается, как сумма каждого из использованных в соединении приборов:

С = С 1 + С 2.

Как определить общее сопротивление произвольного соединения проводников?

То есть такого, в котором последовательные участки сменяют параллельные, и наоборот. Для них по-прежнему справедливы все описанные законы. Только применять их нужно поэтапно.

Сперва полагается мысленно развернуть схему. Если представить ее сложно, то нужно нарисовать то, что получается. Объяснение станет понятнее, если рассмотреть его на конкретном примере (см. рисунок).

Ее удобно начать рисовать с точек Б и В. Их необходимо поставить на некотором удалении друг от друга и от краев листа. Слева к точке Б подходит один провод, а вправо направлены уже два. Точка В, напротив, слева имеет два ответвления, а после нее расположен один провод.

Теперь необходимо заполнить пространство между этими точками. По верхнему проводу нужно расположить три резистора с коэффициентами 2, 3 и 4, а снизу пойдет тот, у которого индекс равен 5. Первые три соединены последовательно. С пятым резистором они параллельны.

Оставшиеся два резистора (первый и шестой) включены последовательно с рассмотренным участком БВ. Поэтому рисунок можно просто дополнить двумя прямоугольниками по обе стороны от выбранных точек. Осталось применить формулы для расчета сопротивления:

  • сначала ту, которая приведена для последовательного соединения;
  • потом для параллельного;
  • и снова для последовательного.

Подобным образом можно развернуть любую, даже очень сложную схему.

Задача на последовательное соединение проводников

Условие. В цепи друг за другом подсоединены две лампы и резистор. Общее напряжение равно 110 В, а сила тока 12 А. Чему равно сопротивление резистора, если каждая лампа рассчитана на напряжение в 40 В?

Решение. Поскольку рассматривается последовательное соединение, формулы его законов известны. Нужно только правильно их применить. Начать с того, чтобы выяснить значение напряжения, которое приходится на резистор. Для этого из общего нужно вычесть два раза напряжение одной лампы. Получается 30 В.

Теперь, когда известны две величины, U и I (вторая из них дана в условии, так как общий ток равен току в каждом последовательном потребителе), можно сосчитать сопротивление резистора по закону Ома. Оно оказывается равным 2,5 Ом.

Ответ. Сопротивление резистора равно 2,5 Ом.

Задача на параллельное и последовательное

Условие. Имеются три конденсатора с емкостями 20, 25 и 30 мкФ. Определите их общую емкость при последовательном и параллельном соединении.

Решение. Проще начать с В этой ситуации все три значения нужно просто сложить. Таким образом, общая емкость оказывается равной 75 мкФ.

Несколько сложнее расчеты будут при последовательном соединении этих конденсаторов. Ведь сначала нужно найти отношения единицы к каждой из этих емкостей, а потом сложить их друг с другом. Получается, что единица, деленная на общую емкость, равна 37/300. Тогда искомая величина получается приблизительно 8 мкФ.

Ответ. Общая емкость при последовательном соединении 8 мкФ, при параллельном — 75 мкФ.

Последовательное соединение сопротивлений

Возьмем три неизменных сопротивления R1, R2 и R3 и включим их в цепь так, чтоб конец первого сопротивления R1 был соединен с началом второго сопротивления R 2, конец второго — с началом третьего R 3, а к началу первого сопротивления и к концу третьего подведем проводники от источника тока (рис. 1 ).

Такое соединение сопротивлений именуется поочередным. Разумеется, что ток в таковой цепи будет во всех ее точках один и тот же.

Рис 1 . Последовательное соединение сопротивлений

Как найти общее сопротивление цепи, если все включенные в нее поочередно сопротивления мы уже знаем? Используя положение, что напряжение U на зажимах источника тока равно сумме падений напряжений на участках цепи, мы можем написать:

U = U1 + U2 + U3

где

U1 = IR1 U2 = IR2 и U3 = IR3

либо

IR = IR1 + IR2 + IR3

Вынеся в правой части равенства I за скобки, получим IR = I(R1 + R2 + R3) .

Поделив сейчас обе части равенства на I , будем совсем иметь R = R1 + R2 + R3

Таким макаром, мы сделали вывод, что при поочередном соединении сопротивлений общее сопротивление всей цепи равно сумме сопротивлений отдельных участков.

Проверим этот вывод на последующем примере. Возьмем три неизменных сопротивления, величины которых известны (к примеру, R1 == 10 Ом, R 2 = 20 Ом и R 3 = 50 Ом). Соединим их поочередно (рис. 2 ) и подключим к источнику тока, ЭДС которого равна 60 В (внутренним сопротивлением источника тока пренебрегаем).

Рис. 2. Пример поочередного соединения 3-х сопротивлений

Подсчитаем, какие показания должны дать приборы, включенные, как показано на схеме, если замкнуть цепь. Определим наружное сопротивление цепи: R = 10 + 20 + 50 = 80 Ом.

Найдем ток в цепи по закону Ома: 60 / 80 = 0 ,75 А

Зная ток в цепи и сопротивления ее участков, определим падение напряжения на каждое участке цепи U 1 = 0,75х 10 = 7,5 В, U 2 = 0,75 х 20=15 В, U3 = 0,75 х 50 = 37,5 В.

Зная падение напряжений на участках, определим общее падение напряжения во наружной цепи, т. е. напряжение на зажимах источника тока U = 7,5+15 + 37,5 = 60 В.

Мы получили таким макаром, что U = 60 В, т. е. несуществующее равенство ЭДС источника тока и его напряжения. Разъясняется это тем, что мы пренебрегли внутренним сопротивлением источника тока.

Замкнув сейчас ключ выключатель К, можно убедиться по устройствам, что наши подсчеты приблизительно верны.

Возьмем два неизменных сопротивления R1 и R2 и соединим их так, чтоб начала этих сопротивлений были включены в одну общую точку а, а концы — в другую общую точку б. Соединив потом точки а и б с источником тока, получим замкнутую электронную цепь. Такое соединение сопротивлений именуется параллельным соединением.

Рис 3. Параллельное соединение сопротивлений

Проследим течение тока в этой цепи. От положительного полюса источника тока по соединительному проводнику ток дойдет до точки а. В точке а он разветвится, потому что тут сама цепь разветвляется на две отдельные ветки: первую ветвь с сопротивлением R1 и вторую — с сопротивлением R2. Обозначим токи в этих ветвях соответственно через I1 и I 2. Любой из этих токов пойдет по собственной ветки до точки б. В этой точке произойдет слияние токов в один общий ток, который и придет к отрицательному полюсу источника тока.

Таким макаром, при параллельном соединении сопротивлений выходит разветвленная цепь. Поглядим, какое же будет соотношение меж токами в составленной нами цепи.

Включим амперметр меж положительным полюсом источника тока (+) и точкой а и заметим его показания. Включив потом амперметр (показанный «а рисунке пунктиром) в провод, соединяющий точку б с отрицательным полюсом источника тока (-), заметим, что прибор покажет ту же величину силы тока.

Означает, сила тока в цепи до ее разветвления (до точки а) равна силе тока после разветвления цепи (после точки б).

Будем сейчас включать амперметр попеременно в каждую ветвь цепи, запоминая показания прибора. Пусть в первой ветки амперметр покажет силу тока I1 , а во 2-ой — I 2. Сложив эти два показания амперметра, мы получим суммарный ток, по величине равный току I до разветвления (до точки а).

Как следует, сила тока, протекающего до точки разветвления, равна сумме сил токов, утекающих от этой точки. I = I1 + I2 Выражая это формулой, получим

Это соотношение, имеющее огромное практическое значение, носит заглавие закона разветвленной цепи .

Разглядим сейчас, каково будет соотношение меж токами в ветвях.

Включим меж точками а и б вольтметр и поглядим, что он нам покажет. Во-1-х, вольтметр покажет напряжение источника тока, потому что он подключен, как это видно из рис. 3 , конкретно к зажимам источника тока. Во-2-х, вольтметр покажет падения напряжений U1 и U2 на сопротивлениях R1 и R2, потому что он соединен с началом и концом каждого сопротивления.

Как следует, при параллельном соединении сопротивлений напряжение на зажимах источника тока равно падению напряжения на каждом сопротивлении.

Это дает нам право написать, что U = U1 = U2 ,

где U — напряжение на зажимах источника тока; U1 — падение напряжения на сопротивлении R1 , U2 — падение напряжения на сопротивлении R2. Вспомним, что падение напряжения на участке цепи численно равно произведению силы тока, протекающего через этот участок, на сопротивление участка U = IR .

Потому для каждой ветки можно написать: U1 = I1R1 и U2 = I2R2 , но потому что U1 = U2, то и I1R1 = I2R2 .

Применяя к этому выражению правило пропорции, получим I1/ I2 = U2 / U1 т. е. ток в первой ветки будет во столько раз больше (либо меньше) тока во 2-ой ветки, во сколько раз сопротивление первой ветки меньше (либо больше) сопротивления 2-ой ветки.

Итак, мы пришли к принципиальному выводу, заключающемуся в том, что при параллельном соединении сопротивлений общий ток цепи разветвляется на токи, назад пропорциональные величинам сопротивлении параллельных веток. По другому говоря, чем больше сопротивление ветки, тем наименьший ток потечет через нее, и, напротив, чем меньше сопротивление ветки, тем больший ток потечет через эту ветвь.

Убедимся в корректности этой зависимости на последующем примере. Соберем схему, состоящую из 2-ух параллельно соединенных сопротивлений R1 и R 2, присоединенных к источнику тока. Пусть R1 = 10 Ом, R2 = 20 Ом и U = 3 В.

Подсчитаем поначалу, что покажет нам амперметр, включенный в каждую ветвь:

I1 = U / R1 = 3 / 10 = 0 ,3 А = 300 мА

I 2 = U / R 2 = 3 / 20 = 0,15 А = 150 мА

Общий ток в цепи I = I1 +I2 = 300 + 150 = 450 мА

Проделанный нами расчет подтверждает, что при параллельном соединении сопротивлений ток в цепи разветвляется назад пропорционально сопротивлениям.

Вправду, R1 == 10 Ом в два раза меньше R 2 = 20 Ом, при всем этом I1 = 300 мА в два раза больше I2 = 150 мА. Общий ток в цепи I = 450 мА разветвился на две части так, что большая его часть (I1 = 300 мА) пошла через наименьшее сопротивление (R1 = 10 Ом), а наименьшая часть (R2 = 150 мА) -через большее сопротивление (R 2 = 20 Ом).

Такое разветвление тока в параллельных ветвях сходно с течением воды по трубам. Представьте для себя трубу А, которая в каком-то месте разветвляется на две трубы Б и В различного поперечника (рис. 4). Потому что поперечник трубы Б больше поперечника трубок В, то через трубу Б в одно и то же время пройдет больше воды, чем через трубу В, которая оказывает сгустку воды большее сопротивление.

Рис. 4

Разглядим сейчас, чему будет равно общее сопротивление наружной цепи, состоящей из 2-ух параллельно соединенных сопротивлений.

Под этим общим сопротивлением наружной цепи нужно осознавать такое сопротивление, которым можно было бы поменять при данном напряжении цепи оба параллельно включенных сопротивления, не изменяя при всем этом тока до разветвления. Такое сопротивление именуется эквивалентным сопротивлением.

Вернемся к цепи, показанной на рис. 3, и поглядим, чему будет равно эквивалентное сопротивление 2-ух параллельно соединенных сопротивлений. Применяя к этой цепи закон Ома, мы можем написать: I = U/R , где I — ток во наружной цепи (до точки разветвления), U — напряжение наружной цепи, R — сопротивление наружной цепи, т. е. эквивалентное сопротивление.

Точно так же для каждой ветки I1 = U1 / R1 , I2 = U2 / R2 , где I1 и I 2 — токи в ветвях; U1 и U2 — напряжение на ветвях; R1 и R2 — сопротивления веток.

По закону разветвленной цепи: I = I1 + I2

Подставляя значения токов, получим U / R = U1 / R1 + U2 / R2

Потому что при параллельном соединении U = U1 = U2 , то можем написать U / R = U / R1 + U / R2

Вынеся U в правой части равенства за скобки, получим U / R = U (1 / R1 + 1 / R2 )

Разделив сейчас обе части равенства на U , будем совсем иметь 1 / R = 1 / R1 + 1 / R2

Помня, что проводимостью именуется величина, оборотная сопротивлению , мы можем сказать, что в приобретенной формуле 1 / R — проводимость наружной цепи; 1 / R1 проводимость первой ветки; 1 / R2- проводимость 2-ой ветки.

На основании этой формулы делаем вывод: при параллельном соединении проводимость наружной цепи равна сумме проводимостей отдельных веток.

Как следует, чтоб найти эквивалентное сопротивление включенных параллельно сопротивлений, нужно найти проводимость цепи и взять величину, ей оборотную.

Из формулы также следует, что проводимость цепи больше проводимости каждой ветки, а это означает, что эквивалентное сопротивление наружной цепи меньше меньшего из включенных параллельно сопротивлений.

Рассматривая случай параллельного соединения сопротивлений, мы взяли более ординарную цепь, состоящую из 2-ух веток. Но на практике могут повстречаться случаи, когда цепь состоит из 3-х и поболее параллельных веток. Как поступать в этих случаях?

Оказывается, все приобретенные нами соотношения остаются справедливыми и для цепи, состоящей из хоть какого числа параллельно соединенных сопротивлений.

Чтоб убедиться в этом, разглядим последующий пример.

Возьмем три сопротивления R1 = 10 Ом, R2 = 20 Ом и R3 = 60 Ом и соединим их параллельно. Определим эквивалентное сопротивление цепи (рис. 5 ). R = 1 / 6 Как следует, эквивалентное сопротивление R = 6 Ом.

Таким макаром, эквивалентное сопротивление меньше меньшего из включенных параллельно в цепь сопротивлений , т. е. меньше сопротивления R1.

Поглядим сейчас, вправду ли это сопротивление является эквивалентным, т. е. таким, которое могло бы поменять включенные параллельно сопротивления в 10, 20 и 60 Ом, не изменяя при всем этом силы тока до разветвления цепи.

Допустим, что напряжение наружной цепи, а как следует, и напряжение на сопротивлениях R1, R2, R3 равно 12 В. Тогда сила токов в ветвях будет: I1 = U/R1 = 12 / 10 = 1 ,2 А I 2 = U/R 2 = 12 / 20 = 1 ,6 А I 3 = U/R1 = 12 / 60 = 0,2 А

Общий ток в цепи получим, пользуясь формулой I = I1 + I2 + I3 =1,2 + 0,6 + 0,2 = 2 А.

Проверим по формуле закона Ома, получится ли в цепи ток силой 2 А, если заместо 3-х параллельно включенных узнаваемых нам сопротивлений включено одно эквивалентное им сопротивление 6 Ом.

I = U / R = 12 / 6 = 2 А

Как лицезреем, отысканное нами сопротивление R = 6 Ом вправду является для данной цепи эквивалентным.

В этом можно убедиться и на измерительных устройствах, если собрать схему с взятыми нами сопротивлениями, измерить ток во наружной цепи (до разветвления), потом поменять параллельно включенные сопротивления одним сопротивлением 6 Ом и опять измерить ток. Показания амперметра и в том и в другом случае будут приблизительно схожими.

На практике могут повстречаться также параллельные соединения, для которых высчитать эквивалентное сопротивление можно проще, т. е. не определяя за ранее проводимостей, сходу отыскать сопротивление.

К примеру, если соединены параллельно два сопротивления R1 и R2 , то формулу 1 / R = 1 / R1 + 1 / R2 можно конвертировать так: 1/R = (R2 + R1) / R1 R2 и, решая равенство относительно R, получить R = R1 х R2 / (R1 + R2 ), т. е. при параллельном соединении 2-ух сопротивлений эквивалентное сопротивление цепи равно произведению включенных параллельно сопротивлений, деленному на их сумму.

Содержание:

Течение тока в электрической цепи осуществляется по проводникам, в направлении от источника к потребителям. В большинстве подобных схем используются медные провода и электрические приемники в заданном количестве, обладающие различным сопротивлением. В зависимости выполняемых задач, в электрических цепях используется последовательное и параллельное соединение проводников. В некоторых случаях могут быть применены оба типа соединений, тогда этот вариант будет называться смешанным. Каждая схема имеет свои особенности и отличия, поэтому их нужно обязательно заранее учитывать при проектировании цепей, ремонте и обслуживании электрооборудования.

Последовательное соединение проводников

В электротехнике большое значение имеет последовательное и параллельное соединение проводников в электрической цепи. Среди них часто используется схема последовательного соединения проводников предполагающая такое же соединение потребителей. В этом случае включение в цепь выполняется друг за другом в порядке очередности. То есть, начало одного потребителя соединяется с концом другого при помощи проводов, без каких-либо ответвлений.

Свойства такой электрической цепи можно рассмотреть на примере участков цепи с двумя нагрузками. Силу тока, напряжение и сопротивление на каждом из них следует обозначить соответственно, как I1, U1, R1 и I2, U2, R2. В результате, получились соотношения, выражающие зависимость между величинами следующим образом: I = I1 = I2, U = U1 + U2, R = R1 + R2. Полученные данные подтверждаются практическим путем с помощью проведения измерений амперметром и вольтметром соответствующих участков.

Таким образом, последовательное соединение проводников отличается следующими индивидуальными особенностями:

  • Сила тока на всех участках цепи будет одинаковой.
  • Общее напряжение цепи составляет сумму напряжений на каждом участке.
  • Общее сопротивление включает в себя сопротивления каждого отдельного проводника.

Данные соотношения подходят для любого количества проводников, соединенных последовательно. Значение общего сопротивления всегда выше, чем сопротивление любого отдельно взятого проводника. Это связано с увеличением их общей длины при последовательном соединении, что приводит и к росту сопротивления.

Если соединить последовательно одинаковые элементы в количестве n, то получится R = n х R1, где R — общее сопротивление, R1 — сопротивление одного элемента, а n — количество элементов. Напряжение U, наоборот, делится на равные части, каждая из которых в n раз меньше общего значения. Например, если в сеть с напряжением 220 вольт последовательно включаются 10 ламп одинаковой мощности, то напряжение в любой из них составит: U1 = U/10 = 22 вольта.

Проводники, соединенные последовательно, имеют характерную отличительную особенность. Если во время работы отказал хотя-бы один из них, то течение тока прекращается во всей цепи. Наиболее ярким примером является , когда одна перегоревшая лампочка в последовательной цепи, приводит к выходу из строя всей системы. Для установления перегоревшей лампочки понадобится проверка всей гирлянды.

Параллельное соединение проводников

В электрических сетях проводники могут соединяться различными способами: последовательно, параллельно и комбинированно. Среди них параллельное соединение это такой вариант, когда проводники в начальных и конечных точках соединяются между собой. Таким образом, начала и концы нагрузок соединяются вместе, а сами нагрузки располагаются параллельно относительно друг друга. В электрической цепи могут содержаться два, три и более проводников, соединенных параллельно.

Если рассматривать последовательное и параллельное соединение, сила тока в последнем варианте может быть исследована с помощью следующей схемы. Берутся две лампы накаливания, обладающие одинаковым сопротивлением и соединенные параллельно. Для контроля к каждой лампочке подключается собственный . Кроме того, используется еще один амперметр, контролирующий общую силу тока в цепи. Проверочная схема дополняется источником питания и ключом.

После замыкания ключа нужно контролировать показания измерительных приборов. Амперметр на лампе № 1 покажет силу тока I1, а на лампе № 2 — силу тока I2. Общий амперметр показывает значение силы тока, равное сумме токов отдельно взятых, параллельно соединенных цепей: I = I1 + I2. В отличие от последовательного соединения, при перегорании одной из лампочек, другая будет нормально функционировать. Поэтому в домашних электрических сетях используется параллельное подключение приборов.

С помощью такой же схемы можно установить значение эквивалентного сопротивления. С этой целью в электрическую цепь добавляется вольтметр. Это позволяет измерить напряжение при параллельном соединении, сила тока при этом остается такой же. Здесь также имеются точки пересечения проводников, соединяющих обе лампы.

В результате измерений общее напряжение при параллельном соединении составит: U = U1 = U2. После этого можно рассчитать эквивалентное сопротивление, условно заменяющее все элементы, находящиеся в данной цепи. При параллельном соединении, в соответствии с законом Ома I = U/R, получается следующая формула: U/R = U1/R1 + U2/R2, в которой R является эквивалентным сопротивлением, R1 и R2 — сопротивления обеих лампочек, U = U1 = U2 — значение напряжения, показываемое вольтметром.

Следует учитывать и тот фактор, что токи в каждой цепи, в сумме составляют общую силу тока всей цепи. В окончательном виде формула, отражающая эквивалентное сопротивление будет выглядеть следующим образом: 1/R = 1/R1 + 1/R2. При увеличении количества элементов в таких цепях — увеличивается и число слагаемых в формуле. Различие в основных параметрах отличают друг от друга и источников тока, позволяя использовать их в различных электрических схемах.

Параллельное соединение проводников характеризуется достаточно малым значением эквивалентного сопротивления, поэтому сила тока будет сравнительно высокой. Данный фактор следует учитывать, когда в розетки включается большое количество электроприборов. В этом случае сила тока значительно возрастает, приводя к перегреву кабельных линий и последующим возгораниям.

Законы последовательного и параллельного соединения проводников

Данные законы, касающиеся обоих видов соединений проводников, частично уже были рассмотрены ранее.

Для более четкого их понимания и восприятия в практической плоскости, последовательное и параллельное соединение проводников, формулы следует рассматривать в определенной последовательности:

  • Последовательное соединение предполагает одинаковую силу тока в каждом проводнике: I = I1 = I2.
  • параллельное и последовательное соединение проводников объясняет в каждом случае по-своему. Например, при последовательном соединении, напряжения на всех проводниках будут равны между собой: U1 = IR1, U2 = IR2. Кроме того, при последовательном соединении напряжение составляет сумму напряжений каждого проводника: U = U1 + U2 = I(R1 + R2) = IR.
  • Полное сопротивление цепи при последовательном соединении состоит из суммы сопротивлений всех отдельно взятых проводников, независимо от их количества.
  • При параллельном соединении напряжение всей цепи равно напряжению на каждом из проводников: U1 = U2 = U.
  • Общая сила тока, измеренная во всей цепи, равна сумме токов, протекающих по всем проводникам, соединенных параллельно между собой: I = I1 + I2.

Для того чтобы более эффективно проектировать электрические сети, нужно хорошо знать последовательное и параллельное соединение проводников и его законы, находя им наиболее рациональное практическое применение.

Смешанное соединение проводников

В электрических сетях как правило используется последовательное параллельное и смешанное соединение проводников, предназначенное для конкретных условий эксплуатации. Однако чаще всего предпочтение отдается третьему варианту, представляющему собой совокупность комбинаций, состоящих из различных типов соединений.

В таких смешанных схемах активно применяется последовательное и параллельное соединение проводников, плюсы и минусы которых обязательно учитываются при проектировании электрических сетей. Эти соединения состоят не только из отдельно взятых резисторов, но и довольно сложных участков, включающих в себя множество элементов.

Смешанное соединение рассчитывается в соответствии с известными свойствами последовательного и параллельного соединения. Метод расчета заключается в разбивке схемы на более простые составные части, которые считаются отдельно, а потом суммируются друг с другом.

Во многих электрических схемах мы можем обнаружить последовательное и . Разработчик схем может, например, объединить несколько резисторов со стандартными значениями (E-серии), чтобы получить необходимое сопротивление.

Последовательное соединении резисторов — это такое соединение, при котором ток, протекающий через каждый резистор одинаков, поскольку имеется только одно направление для протекания тока. В тоже время падение напряжения будет пропорционально сопротивлению каждого резистора в последовательной цепи.

Последовательное соединение резисторов

Пример № 1

Используя закон Ома, необходимо вычислить эквивалентное сопротивление серии последовательно соединенных резисторов (R1. R2, R3), а так же падение напряжения и мощность для каждого резистора:

Все данные могут быть получены с помощью закона Ома и для лучшего понимания представлены в виде следующей таблицы:

Пример № 2

а) без подключенного резистора R3

б) с подключенным резистором R3

Как вы можете видеть, выходное напряжение U без нагрузочного резистора R3, составляет 6 вольт, но то же выходное напряжение при подключении R3 становится всего лишь 4 В. Таким образом, нагрузка, подключенная к делителю напряжения, провоцирует дополнительное падение напряжение. Данный эффект снижения напряжения может быть компенсирован с помощью установленного вместо постоянного резистора, с помощью которого можно скорректировать напряжение на нагрузке.

Онлайн калькулятор расчета сопротивления последовательно соединенных резисторов

Чтобы быстро вычислить общее сопротивление двух и более резисторов, соединенных последовательно, вы можете воспользоваться следующим онлайн калькулятором:

Подведем итог

Когда два или несколько резисторов соединены вместе (вывод одного соединяется с выводом другого резистора) — то это последовательное соединение резисторов. Ток, протекающий через резисторы имеет одно и тоже значение, но падение напряжения на них не одно и то же. Оно определяется сопротивлением каждого резистора, которое рассчитывается по закону Ома (U = I * R).

Если нам надо, чтобы электроприбор работал, мы должны подключить его к . При этом ток должен проходить через прибор и возвращаться вновь к источнику, то есть цепь должна быть замкнутой.

Но подключение каждого прибора к отдельному источнику осуществимо, в основном, в лабораторных условиях. В жизни же приходится иметь дело с ограниченным количеством источников и довольно большим количеством потребителей тока. Поэтому создают системы соединений, позволяющие нагрузить один источник большим количеством потребителей. Системы при этом могут быть сколь угодно сложными и разветвленными, но в их основе лежит всего два вида соединения: последовательное и параллельное соединение проводников. Каждый вид имеет свои особенности, плюсы и минусы. Рассмотрим их оба.

Последовательное соединение проводников

Последовательное соединение проводников – это включение в электрическую цепь нескольких приборов последовательно, друг за другом. Электроприборы в данном случае можно сравнить с людьми в хороводе, а их руки, держащие друг друга – это провода, соединяющие приборы. Источник тока в данном случае будет одним из участников хоровода.

Напряжение всей цепи при последовательном соединении будет равно сумме напряжений на каждом включенном в цепь элементе. Сила тока в цепи будет одинакова в любой точке. А сумма сопротивлений всех элементов составит общее сопротивление всей цепи. Поэтому последовательное сопротивление можно выразить на бумаге следующим образом:

I=I_1=I_2=⋯=I_n ; U=U_1+U_2+⋯+U_n ; R=R_1+R_2+⋯+R_n ,

Плюсом последовательного соединения является простота сборки, а минусом – то, что если один элемент выйдет из строя, то ток пропадет во всей цепи. В такой ситуации неработающий элемент будет подобен ключу в выключенном положении. Пример из жизни неудобства такого соединения наверняка припомнят все люди постарше, которые украшали елки гирляндами из лампочек.

Если в такой гирлянде выходила из строя хотя бы одна лампочка, приходилось перебирать их все, пока не найдешь ту самую, перегоревшую. В современных гирляндах эта проблема решена. В них используют специальные диодные лампочки, в которых при перегорании сплавляются вместе контакты, и ток продолжает беспрепятственно проходить дальше.

Параллельное соединение проводников

При параллельном соединении проводников все элементы цепи подключаются к одной и той же паре точек, можно назвать их А и В. К этой же паре точек подключают источник тока. То есть получается, что все элементы подключены к одинаковому напряжению между А и В. В то же время ток как бы разделяется на все нагрузки в зависимости от сопротивления каждой из них.

Параллельное соединение можно сравнить с течением реки, на пути которой возникла небольшая возвышенность. Вода в таком случае огибает возвышенность с двух сторон, а потом вновь сливается в один поток. Получается островок посреди реки. Так вот параллельное соединение – это два отдельных русла вокруг острова. А точки А и В – это места, где разъединяется и вновь соединяется общее русло реки.

Напряжение тока в каждой отдельной ветви будет равно общему напряжению в цепи. Общий ток цепи будет складываться из токов всех отдельных ветвей. А вот общее сопротивление цепи при параллельном соединении будет меньше сопротивления тока на каждой из ветвей. Это происходит потому, что общее сечение проводника между точками А и В как бы увеличивается за счет увеличения числа параллельно подключенных нагрузок. Поэтому общее сопротивление уменьшается. Параллельное соединение описывается следующими соотношениями:

U=U_1=U_2=⋯=U_n ; I=I_1+I_2+⋯+I_n ; 1/R=1/R_1 +1/R_2 +⋯+1/R_n ,

где I — сила тока, U- напряжение, R – сопротивление, 1,2,…,n – номера элементов, включенных в цепь.

Огромным плюсом параллельного соединения является то, что при выключении одного из элементов, цепь продолжает функционировать дальше. Все остальные элементы продолжают работать. Минусом является то, что все приборы должны быть рассчитаны на одно и то же напряжение. Именно параллельным образом устанавливают розетки сети 220 В в квартирах. Такое подключение позволяет включать различные приборы в сеть совершенно независимо друг от друга, и при выходе их строя одного из них, это не влияет на работу остальных.

Нужна помощь в учебе?

Предыдущая тема: Расчёт сопротивления проводников и реостаты: формулы
Следующая тема:&nbsp&nbsp&nbspРабота и мощность тока

Последовательное соединение резисторов

При последовательном соединении резисторов, на этом участке цепи ток будет все время одинаковым.

Формула расчета простая — Суммарное сопротивление на участке цепи будет равна сумме значений сопротивлений всех резисторов, включенных в цепь. Вид формулы следующий

Rсум=∑R, где R – значение каждого из резисторов в цепи

т. е. Вам надо сложить сопротивление всех резисторов в цепи и их сумма покажет общее сопротивление.

Давайте сделаем шаг в сторону, чтобы проще было понять смысл суммарного сопротивления.

Ток течет по цепи, и визуально его можно представит, как поток воды, который течет сквозь трубу.

Трубы меньшего диаметра — по сути, являются резисторами, которые затрудняют проток и делают его меньше. Например, вы когда-нибудь пробовали потянуть напиток из кружки через очень очень узкую трубочку? Усилий можно делать много, но поток в рот будет все равно слишком маленьким. И чем больше таких разных сужений будет, тем сложнее вам будет тянуть напиток через трубочку. По такому же принципу работают резисторы, уменьшая ток.

Закон Ома гласит, что ток равен напряжению, деленному на сопротивление.

I=U/R

Давайте рассмотрим примеры использования формулы, а заодно изучим понятие падения напряжения.

Пример №1

Дана схема цепи с последовательным сопротивлением трех резисторов.

Источник постоянного тока с напряжением на входе 24 V

Три резистора, соединенных последовательно, с сопротивлениями, 6,3 и 3 Ом соответственно. Т.е. R1=6 Ом, R2=3 Ом, R3=3 Ом

Cхема простейшей цепи постоянного тока с последовательным соединением Резисторов

Определим суммарное сопротивление этой простой цепи:

Согласно нашей формуле, суммарное сопротивление последовательно соединенных транзисторов будет равно сумме всех сопротивлений:

Rсум=6Ом+3Ом+3Ом=12 Ом

Давайте определим ток на участке по формуле выше:

I=Uвх/Rсум

I=24В/12Ом=2А

Таким образом, мы определили, что ток на всем участке будет равен 2 амперам.

Давайте определим падение напряжения на каждом элементе нашей цепи.

Еще небольшое отступление. Каждый резистор будет как бы забирать часть напряжения на себя, и чем выше его сопротивление, тем больше напряжения будет на нем «падать».

В любом замкнутом контуре, сумма напряжений равна нулю. Таким образом, в нашем случае справедлива формула Uвх=Ur1+ Ur2+ Ur3

А как определить эти значения? Очень просто. Мы знаем, что ток везде одинаковый, и знаем значения каждого отдельного сопротивления R1,R2,R3.

Тогда Ur1= R1*I=6Ом*2А=12В

Ur2= R2*I=3Ом*2А=6В

Ur3= R3*I=3Ом*2А=6В

Пример №2

Схема цепи с последовательным сопротивлением двух резисторов.

Схема с двумя резисторами, подключенными последовательно

Источник постоянного тока с напряжением на входе 12 V

Два резистора, соединенных последовательно, с сопротивлениями, 2,1 Ом соответственно. Т.е. R1=2 Ом, R2=1 Ом

Далее все расчеты аналогичны.

Шаг первый — определяем суммарное значение сопротивления: Rсум=2Ом+1Ом=3Ом

Шаг второй — определяем ток в этом участке цепи: I=12В/3Ом=4А

Шаг третий — считаем падения напряжения на каждом элементе цепи с последовательным сопротивлением двух резисторов:

Ur1= R1*I=2Ом*4А=8В

Ur2= R2*I=1Ом*4А=4В

Как и в прошлом расчете, сумма падений напряжений (8+4) будет равна входному напряжению.

Обратите внимание, что в случае последовательного соединения резисторов, суммарное сопротивление всегда будет больше любого отдельно взятого резистора в этой цепи.

Надеюсь, что это поможет вам в понимании. Если есть какие-то вопросы по написанному материалу, то пишите в комментах.

Элеком37, Закон Ома. Последовательное и параллельное соединение проводников.

Закон Ома. Последовательное и параллельное соединение проводников.

Немецкий физик Г.Ом в 1826 году экспериментально установил, что сила тока I, текущего по однородному металлическому проводнику (то есть проводнику, в котором не действуют сторонние силы) сопротивлением R, пропорциональна напряжению U на концах проводника:

Величину R принято называть электрическим сопротивлением. Проводник, обладающий электрическим сопротивлением, называется резистором. Это соотношение выражает закон Ома для однородного участка цепи: сила тока в проводнике прямо пропорциональна приложенному напряжению и обратно пропорциональна сопротивлению проводника.

Проводники, подчиняющиеся закону Ома, называются линейными. Графическая зависимость силы тока I от напряжения U (такие графики называются вольт-амперными характеристиками, сокращенно ВАХ) изображается прямой линией, проходящей через начало координат. Следует отметить, что существует много материалов и устройств, не подчиняющихся закону Ома, например, полупроводниковый диод или газоразрядная лампа. Даже у металлических проводников при достаточно больших токах наблюдается отклонение от линейного закона Ома, так как электрическое сопротивление металлических проводников растет с ростом температуры.

Проводники в электрических цепях можно соединять двумя способами: последовательно и параллельно. У каждого способа есть свои закономерности.

1. Закономерности последовательного соединения:

Формула для общего сопротивления последовательно соединенных резисторов справедлива для любого числа проводников. Если же в цепь последовательно включено n одинаковых сопротивлений R, то общее сопротивление R0 находится по формуле:

2. Закономерности параллельного соединения:

Формула для общего сопротивления параллельно соединенных резисторов справедлива для любого числа проводников. Если же в цепь параллельно включено n одинаковых сопротивлений R, то общее сопротивление R0 находится по формуле:

Электроизмерительные приборы

Для измерения напряжений и токов в электрических цепях постоянного тока используются специальные приборы – вольтметры и амперметры.

Вольтметр предназначен для измерения разности потенциалов, приложенной к его клеммам. Он подключается параллельно участку цепи, на котором производится измерение разности потенциалов. Любой вольтметр обладает некоторым внутренним сопротивлением RB. Для того чтобы вольтметр не вносил заметного перераспределения токов при подключении к измеряемой цепи, его внутреннее сопротивление должно быть велико по сравнению с сопротивлением того участка цепи, к которому он подключен.

Амперметр предназначен для измерения силы тока в цепи. Амперметр включается последовательно в разрыв электрической цепи, чтобы через него проходил весь измеряемый ток. Амперметр также обладает некоторым внутренним сопротивлением RA. В отличие от вольтметра, внутреннее сопротивление амперметра должно быть достаточно малым по сравнению с полным сопротивлением всей цепи.

последовательно соединенных резисторов — Расчет сопротивления — CCEA — Редакция GCSE Physics (Single Science) — CCEA

Ток

При последовательном подключении резисторов ток через каждый резистор одинаков.

Ток одинаков во всех точках последовательной цепи.

В схеме ниже: I S = I 1 = I 2 = I 3

Напряжение В (или разность потенциалов)

При последовательном соединении резисторов сумма всех напряжение (иногда называемое разностью потенциалов) на каждом компоненте равно напряжению на источнике питания.

В приведенной выше схеме:

V S = V 1 + V 2 + V 3

Это просто форма закона сохранения энергии .

Напряжение питания — это мера энергии, подводимой к каждому электрону.

Напряжение на каждом компоненте — это электрическая энергия, преобразованная каждым компонентом.

Следовательно, поданная энергия равна преобразованной энергии — энергия не была создана или разрушена в цепи.

В последовательной цепи напряжение на источнике питания равно сумме напряжений на каждом компоненте.

Сопротивление

Общее сопротивление R двух или более резисторов, соединенных последовательно, является суммой отдельных сопротивлений резисторов.

Для схемы выше общее сопротивление R определяется по формуле:

R = R 1 + R 2 + R 3

Пример

Найдите полное сопротивление схемы выше.

Ответ

Это последовательная цепь, поэтому полное сопротивление определяется по формуле:

R = R 1 + R 2 + R 3 + R 4

R = \ ({ 4} \ Omega + {8} \ Omega + {2} \ Omega + {12} \ Omega \)

R = \ ({26} \ Omega \)

Общее сопротивление цепи резисторов равно \ ( {26} \ Omega \). Это означает, что четыре отдельных резистора можно заменить одним резистором из \ ({26} \ Omega \).

Последовательное добавление резисторов всегда увеличивает общее сопротивление.

Ток должен проходить через каждый резистор по очереди, поэтому добавление дополнительного резистора увеличивает уже встреченное сопротивление.

Параллельные резисторы

Ток

При параллельном подключении резисторов ток от источника питания равен сумме токов, протекающих через каждую ветвь цепи.

Другими словами, токи в ветвях параллельной цепи складываются в ток питания.

В приведенной выше схеме:

I S = I 1 + I 2 + I 3

Это соотношение выражает закон сохранения заряда.

Все электроны, вышедшие из источника питания, должны вернуться в источник питания, и каждый электрон может пройти только через одну параллельную ветвь.

В параллельной цепи ток от источника питания равен сумме токов в каждой ветви цепи.

Напряжение

В параллельной цепи напряжение на каждой ветви схемы равно напряжению питания.

Для схемы выше:

В S = В 1 = В 2 = В 3

В параллельной цепи напряжение на каждой ветви равно напряжению питания.

Сопротивление

При параллельном подключении резисторов общее сопротивление R рассчитывается по формуле:

\ [\ frac {1} {R} = \ frac {1} {R} _ {1} + \ frac {1} {R} _ {2} + \ frac {1} {R} _ {3} \]

Сопротивление серии

Сопротивление серии
(Rs) и его коэффициенты Три различных фактора вызывают сопротивление серии в солнечных элементах: -Текущее движение через эмиттер и базу солнечного элемента -Контактное сопротивление между кремнием и металлическим контактом -Сопротивление задних и верхних металлических контактов.Ключевым эффектом Rs является уменьшение коэффициента заполнения, даже несмотря на то, что чрезвычайно высокие значения могут также снизить ток короткого замыкания. I = I L -I 0 ехр [q (V + IRS) / nkT] куда: I: выходной ток ячейки I L : световой ток В : напряжение на клеммах ячейки ’ T : температура n : коэффициент идеальности вопросы и ответы: константы R S : последовательное сопротивление ячейки.
Формулы для последовательного сопротивления и FF Формула напоминает неявную функцию, основанную на появлении тока ( I) , с двух сторон уравнения, и для его решения требуются численные методы. Он не влияет на солнечный элемент при напряжении холостого хода, так как общий ток протекает через солнечный элемент и, как следствие, через нулевое значение Rs. Но вблизи напряжения холостого хода Rs сильно влияет на ВАХ .Четкий метод оценки Rs солнечного элемента — это найти наклон ВАХ в точке напряжения холостого хода. Уравнение для коэффициента заполнения как функции от Rs может быть представлено с учетом того, что для нормальных значений последовательного сопротивления максимальная мощность может быть аппроксимирована как мощность при отсутствии последовательного сопротивления минус потерянная мощность в последовательном сопротивлении. Уравнение максимальной мощности солнечного элемента составляет: P ‘ MP ≈ V MP I MP — I 2 MP / R s = V MP I MP (1 — (I MP / V MP ) R с ) = P MP (1 — (I SC / V OC ) R с ) P ’ MP = P MP (1 — R S / R CH ) В то время как нормализованное последовательное сопротивление определяется как: r s = R s / R ch Таким образом, следующее уравнение аппроксимирует влияние Rs на выходную мощность солнечного элемента, как определено ниже: P ’ MP = P MP (1 — r S ) Если предположить, что Rs не влияет на напряжение холостого хода и ток короткого замыкания, Rs влияет на FF: P ’ MP = P MP (1 — r S ) V ’ OC I’ SC FF ’= V OC I SC FF (1– r s ) FF ’= FF (1– r S ) В приведенном выше уравнении FF, который не изменяется последовательным сопротивлением, равен FF 0 и FF ‘обозначается FF S .Тогда уравнение выглядит следующим образом: FFs = FF 0 (1– r S ) Тогда немного более точное эмпирическое уравнение для связи FF 0 и FF S : FFs = FF 0 (1–1,1 r S ) + r s 2 / 5,4 Это действительно для r s меньше 0,4 и v oc больше 10 . Уровни тока в любом солнечном элементе имеют огромное влияние на потери от последовательного сопротивления.Сопротивление серии

— Inst Tools

Общее сопротивление в последовательной цепи равно сумме всех частей этой цепи, как показано в уравнении ниже.

R
T = R1 + R2 + R3… и т. Д.

, где
R T = общее сопротивление
R1, R2 и R3 = последовательное сопротивление

Пример:

Последовательная цепь включает последовательно включенные резисторы 60 Ом, 100 Ом и 150 Ом (Рисунок 18). Какое полное сопротивление цепи?

Рисунок 18 Сопротивление в последовательной цепи

Решение:

РТ = R1 + R2 + R3

РТ = 60 + 100 + 150

RT = 310 Ом

Общее напряжение в последовательной цепи равно сумме напряжений на каждом резисторе в цепи (рисунок 19), как показано в уравнении ниже.

VT = V1 + V2 + V 3… и т. Д.

где

VT = общее напряжение

V1 = напряжение на R1

V2 = напряжение на R2

В3 = напряжение на R3

Рисунок 19 Падения напряжения в последовательной цепи

Теперь закон

Ома может применяться ко всей последовательной цепи или к отдельным ее составным частям. При использовании на отдельных компонентах напряжение на этой части равно току, умноженному на сопротивление этой части.Для схемы, показанной на рисунке 20, напряжение можно определить, как показано ниже.

V1 = IR1
V2 = IR2
V3 = IR3

VT = V1 + V2 + V3

VT = 10 вольт + 24 вольт + 36 вольт
VT = 70 вольт

Рисунок 20 Суммарное напряжение в последовательной цепи

Чтобы найти полное напряжение в последовательной цепи, умножьте ток на общее сопротивление, как показано в уравнении ниже.

В
Т = I. R Т

где

VT = общее напряжение
I = ток
R T = общее сопротивление

Пример 1:

Последовательная цепь включает последовательно включенные резисторы 50 Ом, 75 Ом и 100 Ом (Рисунок 21).Найдите напряжение, необходимое для получения тока 0,5 ампер.

Рисунок 21 Пример 1 последовательной цепи

Решение:

Шаг 1: Найдите ток в цепи. Как мы уже знаем, ток в последовательной цепи один и тот же, он уже составляет 0,5 ампер.

Шаг 2: Найдите R T

РТ = R1 + R2 + R3

RT = 50 Ом + 75 Ом + 100 Ом

RT = 225 Ом

Шаг 3: Найдите VT.

Используйте закон Ома. V T = I. R Т

VT = 0,5 x 225

VT = 112,5 вольт

Пример 2:

Батарея на 120 В соединена последовательно с тремя резисторами: 40 Ом, 60 Ом и 100 Ом (Рисунок 22). Найдите напряжение на каждом резисторе.

Рисунок 22 Пример 2 последовательной цепи

Решение:

Шаг 1: Найдите полное сопротивление.

РТ = R1 + R2 + R3

RT = 40 Ом + 60 Ом + 100 Ом

RT = 200 Ом

Шаг 2: Найдите ток в цепи (I).

Используйте закон Ома. V T = I. R Т

Решение для I

I = V T / R T

I = 120/200 = 0,6 ампер

Шаг 3: Найдите напряжение на каждом компоненте.

V1 = IR1
V1 = (0,6 А) (40 Ом)
V1 = 24 В

V2 = IR2
V2 = (0,6 А) (60 Ом)
V2 = 36 В

V3 = IR3
V3 = (0,6 А) (100 Ом)
V3 = 60 В

Напряжения V1, V2 и V3 в Примере 2 известны как «падения напряжения» или «падения IR».Их эффект заключается в уменьшении доступного напряжения, подаваемого на другие компоненты схемы. Сумма падений напряжения в любой последовательной цепи всегда равна приложенному напряжению. Мы можем проверить наш ответ в примере 2, используя уравнение

ниже.
VT = V1 + V2 + V3

120 В = 24 В + 36 В + 60 В

120 вольт = 120 вольт

Что такое эквивалентное последовательное сопротивление (ESR)?

Что такое эквивалентное последовательное сопротивление?

Эквивалентное последовательное сопротивление конденсатора — это внутреннее сопротивление, которое появляется последовательно с емкостью устройства.Почти все конденсаторы проявляют это свойство в разной степени в зависимости от конструкции, диэлектрических материалов, качества и надежности конденсатора. Значения эквивалентного последовательного сопротивления (ESR) варьируются от нескольких миллиом до нескольких Ом и приводят к потерям мощности, снижению эффективности и нестабильности цепей источников питания и регуляторов.

Источник изображения

Алюминиевые электролитические конденсаторы и танталовые конденсаторы имеют более высокое значение ESR, чем керамические конденсаторы той же емкости и номинального напряжения.Конденсаторы из полипропилена и полиэстера находятся между ними, но обычно не используются в импульсных источниках питания из-за их больших физических размеров.

Основные части ESR

  • Металлическое сопротивление
  • Электролитическое сопротивление и сопротивление бумаги, зависящее от частоты и температуры
  • Диэлектрик, зависящий от частоты

Факторы, увеличивающие значение СОЭ

  1. Плохие электрические соединения; — Соединение между медными выводами и алюминиевыми пластинами в конденсаторе обычно выполняется сваркой или механическим обжимом.Этот тип соединений вносит некоторое последовательное сопротивление и используется, потому что алюминий не может быть припаян.
  2. Сушка раствора конденсаторного электролита. По мере высыхания жидкого компонента электролита из-за повышенных температур электрическое сопротивление увеличивается.
  3. СОЭ увеличивается с увеличением температуры и частоты. В источниках питания с большими токами рассеиваемая мощность, связанная с ESR, может еще больше повысить температуру и привести к выходу конденсатора из строя.

Влияние частоты на СОЭ

ERS — это часть импеданса конденсатора, которая вызывает общие потери реальной мощности. Это зависит от частоты, как видно из приведенного ниже уравнения:

Где DFR — коэффициент рассеяния, связанный с контактным сопротивлением, DFL — с потерями на утечку, а DFD — с диэлектрическими потерями.

Сверху утечка и диэлектрические потери уменьшаются с увеличением частоты до тех пор, пока контактное сопротивление не станет преобладающим до определенной точки.За пределами этой точки СОЭ становится очень высоким на более высоких частотах, в основном из-за скин-эффекта переменного сигнала.

Минимизация ESR в цепях

  • В высокопроизводительных приложениях используются конденсаторы с низким ESR, такие как твердотельные полимерные конденсаторы с низким ESR, танталовые конденсаторы и многослойные керамические конденсаторы (MLCC).
  • Конденсаторы подключаются параллельно в таких местах, как цепи сглаживания источника питания. Конденсаторы малой емкости подключаются параллельно, а не один большой конденсатор.Это снижает эффективное ESR в дополнение к снижению пульсаций напряжения и позволяет схеме выдерживать более высокие токи с меньшими потерями.

Параллельное подключение конденсаторов

Источник изображения

Производители конденсаторов предоставляют графики ESR в определенном частотном диапазоне, и можно легко определить ESR на заданной частоте. Иногда в них не указывается ESR, а вместо этого указывается коэффициент рассеяния. В таком случае СОЭ рассчитывается по формуле:

Где, DF — это общий коэффициент рассеяния всех элементов потерь конденсатора.

Измерение эквивалентного последовательного сопротивления

Измерители ESR используются для измерения последовательного сопротивления в цепи или вне цепи. Во время измерения некоторые измерители сначала производят контролируемый разряд заряженных конденсаторов перед измерением ESR и емкости.

ERS обычно выражается как максимальное значение при 120 Гц и 100 кГц для танталовых и алюминиевых электролитических конденсаторов и при 100 кГц для пленочных конденсаторов.

Преимущества конденсаторов с низким ESR

Конденсаторы с низким ESR имеют то преимущество, что сводят к минимуму потери в конденсаторах, повышают эффективность и стабильность источника питания при одновременном снижении выходного напряжения пульсаций.Некоторые характеристики, которые приводят к более низкому ESR, включают большую емкость, низкий коэффициент рассеяния и низкое напряжение на конденсаторах.

Резисторы в серии Formula

В электрических цепях часто можно заменить группу резисторов одним эквивалентным резистором. Эквивалентное сопротивление ряда последовательно включенных резисторов является суммой значений отдельных сопротивлений. Единицей измерения сопротивления является Ом (Ом), который равен Вольт на Ампер (1 Ом = 1 В / А). Также распространены более крупные резисторы с сопротивлением килоом (1 кОм = 10 3 Ом) или мегаом (1 МОм = 10 6 Ом).

Эквивалентное сопротивление = резистор 1 + резистор 2 + резистор 3 + …

R eq = эквивалентное сопротивление (Ом или более единицы)

R 1 = сопротивление первого резистора (Ом)

R 2 = сопротивление второго резистора (Ом)

R 3 = сопротивление третьего резистора (Ом)

Последовательные резисторы Формула Вопросы:

1) Какое сопротивление эквивалентно 480,0 кОм, 320.0 кОм, и резистор 100,0 кОм, подключенный последовательно?

Ответ: Все сопротивления выражены в килоомах, поэтому нет необходимости изменять их единицы. Эквивалентное сопротивление можно найти в кОм по формуле:

.

Эквивалентное сопротивление последовательно соединенных резисторов 480,0 кОм, 320,0 кОм и 100,0 кОм составляет 900,0 кОм.

2) Два резистора включены последовательно в электрическую цепь. Их сопротивления 240.0 кОм и 8,00 МОм. Какое эквивалентное сопротивление?

Ответ: Значения сопротивления выражены в разных единицах измерения. Первым шагом к нахождению эквивалентного сопротивления является преобразование их в общую единицу. Одно из значений может быть преобразовано в ту же единицу, что и другое. В этом решении значения будут преобразованы в мегаом.

Если R 1 = 240,0 кОм и R 2 = 8,00 МОм, то:

R 1 = 240,0 кОм

Эквивалентные сопротивления теперь можно найти по формуле:

Сопротивление, эквивалентное 240.Последовательные резисторы 0 кОм и 8,00 МОм составляют 8,24 МОм.

Ресурсы

Последовательная цепь

Глобусы, соединенные последовательно

В последовательной цепи одна за другой подключены две или более нагрузки.

У тока есть только один путь, по которому оно может течь.

Примером последовательной схемы является набор огней на елку. Все шары ставятся один за другим.

Путь только один, поэтому ток будет одинаковым в любой точке цепи.

Принципиальная схема, показывающая три последовательно включенных резистора

Общее сопротивление в последовательной цепи будет равно сумме каждого отдельного сопротивления в цепи.

Чем больше нагрузок помещено в цепь, тем больше сопротивление.

Общее сопротивление для последовательной цепи рассчитывается по следующей формуле:

R T = R 1 + R 2 + R 3

Закон напряжения Кирхгофа

Вольтметр на каждом резисторе в последовательной цепи t

Закон Кирхгофа расширяет закон Ома в отношении напряжений на сопротивлениях в последовательной цепи.Общее напряжение питания будет равно сумме падений напряжения на каждом резисторе.

Общее падение напряжения (В T ) рассчитывается по формуле:

V T = V 1 + V 2 + V 3

Если известны как ток, так и каждое значение сопротивления, то можно использовать закон Ома для расчета падения напряжения на каждом резисторе.

Например:

В 1 = IR 1

Рассеиваемая мощность

Мощность, рассеиваемая в последовательной цепи, зависит от напряжения питания, приложенного к цепи, и тока, протекающего в цепи.Ток зависит от общего сопротивления цепи.

Из раздела о мощности вы знаете, что формула рассеиваемой мощности:

P = VI

Мощность, рассеиваемая в каждом отдельном компоненте, зависит от сопротивления компонента. Общая рассеиваемая мощность будет равна сумме мощности, рассеиваемой каждым отдельным сопротивлением. В зависимости от известных значений комбинации формулы мощности, а также закона Ома могут использоваться для расчета рассеиваемой мощности (или любого другого неизвестного значения).

Пример

На приведенной выше принципиальной схеме, если значения:

В Т = 20 В

R 1 = 50 Ом

R 2 = 20 Ом

R 3 = 100 Ом

Общее сопротивление можно рассчитать следующим образом:

R T = R 1 + R 2 + R 3

р т = 50 + 20 + 100

R T = 170 Ом

Какая общая рассеиваемая мощность?

Вы можете рассчитать текущий расход, а затем рассчитать мощность.Вместо этого вы можете использовать подстановку, чтобы получить формулу.

В формуле P = VI замените I на V T / R T , чтобы получить формулу

P T = V T x V T / R T , что совпадает с

P T = V T 2 / R T

P T = 20 2 /170

P T = 0,235 Вт или 235 мВт

Расчет последовательного сопротивления — Pi My Life Up

В этом руководстве мы покажем вам, как рассчитать сопротивление последовательно подключенных резисторов.

Считается, что резисторы находятся в «серии», когда они подключаются друг к другу в одну линию.

Величина тока, протекающего через серию резисторов, остается прежней. Поскольку все резисторы подключаются друг к другу, каждый дополнительный резистор в серии увеличивает общее сопротивление.

В этом отличие от резисторов, включенных параллельно, где ток увеличивается, а сопротивление уменьшается.

Калькулятор сопротивления серии

Несмотря на то, что вычисление общего сопротивления, вносимого последовательными резисторами, является простым процессом, мы включили удобный калькулятор, чтобы сделать вашу жизнь еще проще.

Чтобы использовать этот калькулятор последовательного сопротивления, все, что вам нужно сделать, это ввести количество резисторов и значение каждого резистора.

Общее сопротивление, обеспечиваемое вашими последовательными резисторами, будет автоматически рассчитываться при заполнении каждого текстового поля.

Расчет общего сопротивления последовательно подключенных резисторов

Здесь мы включили основную принципиальную схему, показывающую, как обычно будут выглядеть последовательно включенные резисторы. Эта диаграмма должна дать вам общее представление о том, на что следует обращать внимание в цепи.

В отличие от резисторов, включенных параллельно, вычисление общего сопротивления, обеспечиваемого последовательными резисторами, является очень простым процессом.

Уравнение для расчета сопротивления последовательно соединенных резисторов невероятно простое и буквально представляет собой значение резистора 1 плюс значение резистора 2 плюс значение резистора 3 и так далее.

Несмотря на свою простоту, мы проведем вас через пример расчета общего сопротивления резисторов последовательно.

Использование формулы последовательного сопротивления

В этом разделе мы будем предполагать, что у вас есть четыре последовательно соединенных резистора.

Сопротивление одного резистора 100 Ом , второго 400 Ом , другого 150 Ом и конечного резистора 500 Ом .

Разное

Добавить комментарий

Ваш адрес email не будет опубликован. Обязательные поля помечены *